Muisto läheisestä yhteydestä

31.8.2021 klo 19.38, kirjoittaja
Kategoriat: Kosmokseen kirjoitettua , Kosmologia

Kosminen mikroaaltotausta on vanhinta valoa. Se irtosi aineesta, kun maailmankaikkeus oli 380 000 vuoden ikäinen. Sitä ennen lämpötila oli niin korkea, että atomit eivät pysyneet kasassa, vaan aine koostui irrallisista ytimistä ja elektroneista. Valo poukkoili jatkuvasti niiden sähkövarauksista, eikä päässyt liikkumaan vapaasti. Maailmankaikkeus oli tämän takia läpinäkymätön, kuten Aurinko nyt.

Kun katsoo Aurinkoa, näkee millainen se oli kahdeksan minuuttia sitten, kun silmiimme saapuva valo irtosi sen pinnalta. Kun katsoo kosmista mikroaaltotaustaa, näkee millainen maailmankaikkeus oli 14 miljardia vuotta sitten, kun havaintolaitteeseemme saapuva valo irtosi aineesta.

Vaikka emme näe Auringon sisään, sen pinnalta tulevasta valosta voi tehdä johtopäätöksiä siitä mitä sisustassa tapahtuu. Samoin kosmisesta mikroaaltotaustasta voi lukea jotain siitä, mitä tapahtui ennen sen irtoamista aineesta.

Mikroaaltotaustasta näytetään useimmiten kuva, missä on sen kirkkaus eri puolilla taivasta.

Kosminen mikroaaltotausta eri suunnissa. (Kuva: ESAn Planck-tutkimusryhmä.)

Tällaiset kuvat on koostettu muutamalla aallonpituudella tehdyistä mittauksista. Mikroaaltotaustaa voi myös mitata toisin päin: monella eri aallonpituudella, mutta välittämättä siitä mistä suunnasta valo tulee. Sitä, paljonko valoa on kullakin aallonpituudella kutsutaan spektriksi.

Kun COBE-satelliitin käänteentekevät tulokset mikroaaltotaustasta julkistettiin vuonna 1992, paljon huomiota kiinnitettiin juuri spektriin, jonka mittaus palkittiin vuonna 2006 puolikkaalla Nobelilla. Valon aallonpituus määräytyy suoraan sen energiasta, ja havaittu spektri noudattaa ennustettua lämpötasapainon energiajakaumaa mittauksen tarkkuudella, joka on yksi kymmenestuhannesosa.

Kosminen mikroaaltotausta eri aallonpituuksilla. (Kuva: NASAn COBE-tutkimusryhmä.)

Tämä oli merkittävä todiste sen puolesta, että maailmankaikkeus oli varhain kuuma ja tiheä. On nimittäin vaikea selittää miten mikään tähdissä, molekyylipilvissä tai missään muualla nykymaailmankaikkeudessa syntynyt valo olisi tarkkaan samassa lämpötilassa kaikkialla alun perin, ja koska valo vuorovaikuttaa itsensä kanssa vain heikosti, irrallaan kulkeva valo ei kulje kohti lämpötasapainoa. Mutta vanha valo kantaa yhä muistoa ajasta, jolloin se oli läheisesti yhteydessä aineeseen.

Sittemmin tärkeämmäksi on noussut COBEn mittaus mikroaaltotaustan jakaumasta taivaalla. Tämä ensimmäinen havainto kosmisen mikroaaltotaustan epätasaisuuksista palkittiin toisella puolikkaalla Nobelista vuonna 2006, ja on usein mainittu ”täsmäkosmologian” alkuna. Epätasaisuuksista on tullut kosmologian keskeinen havaintokohde, jonka avulla on onnistuneesti luodattu niin pimeää ainetta kuin muinaisten aikojen inflaatiota.

Mutta myös mikroaaltotaustan aallonpituusjakauma kertoo menneestä. Niin kauan kuin hiukkaset ovat tiukasti kytköksissä toisiinsa, ne siirtyvät takaisin kohti lämpötasapainoa jos niitä häiritään. Palautuminen kestää kuitenkin aikansa, ja valon irrottua aineesta se ei enää juuri vuorovaikuta, ja säilyttää silloisen tilansa. Niinpä mikroaaltotaustan spektriin kirjattu historiaa ajalta hieman ennen valon ja aineen irtoamista.

Jos kosmista keittoa häiritsee ennen kuin maailmankaikkeus on noin vuoden ikäinen, niin se ehtii palautua tasapainoon ennen valon irtoamista 380 000 vuoden iässä. Vuoden ja 380 000 vuoden välillä tapahtuneet jäljet eivät sen sijaan ehdi kadota ennen fossiloitumistaan mikroaaltotaustaan.

Yksi esimerkki mahdollisesta häiriötekijästä on hiukkasten hajoaminen. Jos on olemassa tuntemattomia hiukkasia, joiden elinikä on jotain kuukauden ja miljoonan vuoden väliltä, niiden hajoaminen jättäisi jälkensä mikroaaltotaustaan. COBEn mittausten perusteella tällaisten hiukkasten osuus maailmankaikkeuden energiatiheydestä pitää olla alle kymmenestuhannesosa, koska mitään merkkejä niistä ei ole nähty. Havainnot rajoittavat yhtä lailla myös sitä, paljonko kevyitä mustia aukkoja voi olla olemassa, jotta niiden Hawkingin säteilyn vaikutus ei näkyisi spektrissä.

Tällaisten spekulaatioiden lisäksi on yksi asia, jonka varmasti tiedämme hämmentävän muinaista puuroa: mikroaaltotaustassa näkyvät aineen epätasaisuudet. Gravitaation takia sopan sattumat tihentyvät entisestään, kunnes mukana kasautuvan valon paine työntää tiheän alueen takaisin, minkä jälkeen se taas tihentyy. Tämä aaltoilu työntää keittoa pois lämpötasapainosta siirtämällä energiaa fotonien välillä. Spektriin jää näin jälki keiton lyhyistä aalloista, jotka ovat jo ehtineet vaimeta mikroaaltotaustan irrotessa aineesta ja joita ei siksi siinä suoraan nähdä.

Koska epätasaisuudet ovat hyvin pieniä, sadastuhannesosan suuruisia, COBEn tarkkuus ei riittänyt havaitsemaan niistä johtuvia häiriöitä. Mikroaaltotaustan spektriä ei ole mitattu COBEn jälkeen, ja teknologia on kehittynyt valtavasti kolmen vuosikymmenen aikana, joten nykyisillä laitteilla nämä epätasaisuuksien jäljet näkyisivät. Tähän kaavailtiin 2010-luvulla PIXIE-satelliittia, jota ei kuitenkaan rahoitettu, mutta yhä valmistellaan paluuta spektrin mittaamiseen, jotta näkisimme muinaisten aikojen aineeseen pintaa syvemmälle.

16 kommenttia “Muisto läheisestä yhteydestä”

  1. Lentotaidoton sanoo:

    ”Tähän kaavailtiin 2010-luvulla PIXIE-satelliittia, jota ei kuitenkaan rahoitettu, mutta yhä valmistellaan paluuta spektrin mittaamiseen, jotta näkisimme muinaisten aikojen aineeseen pintaa syvemmälle.”

    Tutkailin noita PIXIE-satelliitin esittelyjä. Niiden mukaisesti laite todella olisi ollut erittäin edistyksellinen (ja olisi ilmeisesti tarkentunut moni asia varhaiskosmologiassa). Kompastuiko vain rahaan, vai oliko muita (teknisiä, tieteellisiä) esteitä? Olettaisi vielä kymmenessä lisävuodessa tulleen paljon lisätietoutta havainnointiin. Tämä laitehan olisi tietysti pureutunut vielä varhaisempiinkin aikoihin eli inflaation kosmologiaan (polarisaatio), tappoiko BICEP2 sähläys rahoituksen?

    1. Syksy Räsänen sanoo:

      En valitettavasti tiedä miksi PIXIEtä ei rahoitettu. Pitääkin kysyä asiasta kun törmään taas spektrimittausten asiantuntijaan.

  2. Arto sanoo:

    Voisiko olla niin, että valon dualismismissa olisi lopulta kyse ”vene vesillä -ilmiöstä. Kun kohdistetaan huomio tarkasti” veneeseen,” niin nähdään” vene”, mutta ”kauempaa katsottuna havaitaan vain liikkuvan veneen muodostamat aallot” https://physicsworld.com/wave-particle-duality-quantified-for-the-first-time/

    1. Syksy Räsänen sanoo:

      Edistys fysiikassa edellyttää teorioiden matemaattisen rakenteen ymmärtämistä. Sitä ei voi saavuttaa tuollaisia populaareja vertauskuvia pohtimalla.

      Aaltohiukkasdualismista hieman alla. Koska tämä ei liity merkinnän aiheeseen, niin ei siitä sen enempää.

      https://www.ursa.fi/blogi/kosmokseen-kirjoitettua/pikkuhyrrien-kertomaa/

  3. Cargo sanoo:

    ”Yksi esimerkki mahdollisesta häiriötekijästä on hiukkasten hajoaminen. Jos on olemassa tuntemattomia hiukkasia, joiden elinikä on jotain kuukauden ja miljoonan vuoden väliltä, niiden hajoaminen jättäisi jälkensä mikroaaltotaustaan.”

    Paljonkohan tuossa alkukeitossa on tapahtunut kuplintaa aiheuttaneita satunnaisia fuusiopamauksia, kun ympäristön paine on puristanut alkeishiukkasia kasaan? Lisäksi tuollainen aaltoilu voisi summautua isoksi ’superaalloksi’, jolla saattaa olla kosmisia seurauksia 🙂

    1. Syksy Räsänen sanoo:

      Fuusio tarkoittaa kevyiden atomiydinten yhtymistä raskaammiksi ytimiksi. Tätä tapahtuu vain noin kolmen minuutin ja puolen tunnin välillä: sitä ennen on niin kuuma, että ytimet eivät pysy kasassa, sen jälkeen niin kylmä, että ydinreaktiot sammuvat. Kuplimisella ei ole asian kanssa mitään tekemistä.

  4. Jani sanoo:

    Täyttiko aine tuossa 380000 vuotta vanhassa maailmankaikkeudessa koko maailmankaikkeuden? Valo varmaankin lähti tuolloin joka suuntaan niin valohan etenee ainetta nopeammin niin laajentaako maailman kaikkeuden reunalta lähtenyt valo maailmankaikkeutta?

    1. Syksy Räsänen sanoo:

      Näkyvän aineen ja valon tiheys on sama kaikkialla noin sadastuhannesosan tarkkuudella, pimeän aineen noin tuhannesosan.

      Maailmankaikkeudella ei ole reunaa, ks

      http://www.tiede.fi/artikkeli/blogit/maailmankaikkeutta_etsimassa/rajaton_kasvu

      https://www.ursa.fi/blogi/kosmokseen-kirjoitettua/muotoja-ilman-mittanauhaa/

      1. Martti V sanoo:

        CMB syntyaikoina universumi oli tasaista puuroa, jossa alkoi atomit muodostua. Toki fotonit sinkoilivat mm. atomeista joka suuntaan. Materian määrä universumissa ei varmaankaan ole ääretön? Näin ollen voisi ajatella, että fotonit ovat levinneet laajemmalle kuin materia.

        1. Syksy Räsänen sanoo:

          Emme tiedä onko maailmankaikkeus äärellinen vai ääretön, emmekä näin ollen myöskään sitä, onko ainetta äärettömästi. Tällä ei kuitenkaan ole mitään tekemistä sen kanssa, ovatko fotonit levinneet laajemmalle kuin aine.

          1. Martti V sanoo:

            Olipa universumin topologia mikä tahansa sen tila kasvaa koko ajan. Jos koko on ääretön se kasvaa äärettömästä vielä suuremmaksi eli äärettömäksi. Voidaanko olettaa että taustasäteily on jakautunut tasaisesti tässä tapauksessa?

          2. Syksy Räsänen sanoo:

            Kaikkialla näkemässämme maailmankaikkeudessa kosminen mikroaaltotausta -kuten kaikki muukin- on tilastollisesti samanlaista kaikkialla. (Avaruuden laajeneminen ei tee kosmisesta mikroaaltotaustasta erilaista eri puolilla.) Näemme 50 miljardin valovuoden päähän. Emme tiedä millaista kauempana on, mutta ei ole mitään syytä odottaa, etteikö kosminen mikroaaltotausta olisi tasainen muuallakin.

          3. Martti V sanoo:

            Kiitos vastauksesta. Jos universumi on ääretön, oli alussa myös ääretön lämpötila äärettömän tiheässä. Miksi taustasäteilyn lämpötila on laskenut arvoonsa? Vaikka ääretöntä jakaa millä tahansa tilavuusyksikölle lopputulos on ääretön.

          4. Syksy Räsänen sanoo:

            Sillä, onko maailmankaikkeus äärellinen vai ääretön, ei ole mitään tekemistä sen kanssa, mikä siinä olevan aineen lämpötila on. Tämä riittäköön tästä.

  5. Antti sanoo:

    Muistuuko mieleen onko taustasäteilystä menossa mitään uudenlaisia tutkimuksia vai onko siitä
    kaavittu jo kaikki irti?

    1. Syksy Räsänen sanoo:

      Merkinnässä kirjoitan siitä, miten kosmisen taustasäteilyn spektristä on tehty vain yksi mittaus, ja sitä haluttaisiin mitata tarkemmin.

      Muut kosmisen mikroaaltotaustan suunnitteilla olevat kokeet tähtäävät sen polarisaation tarkempaan mittaamiseen gravitaatioaaltojen jäljen havaitsemiseksi.

      Toistaiseksi varmistuneita kokeita ovat Simons Observatory ja japanilainen LiteBIRD-satelliitti, suunnitteilla on myös nimellä S4 kulkeva maanpäällinen laitteisto.

      http://litebird.jp/eng/

      https://simonsobservatory.org/

      https://cmb-s4.org/

Vastaa

Sähköpostiosoitettasi ei julkaista. Pakolliset kentät on merkitty *


Kuratoitu läpileikkaus

9.8.2021 klo 21.53, kirjoittaja
Kategoriat: Kosmokseen kirjoitettua , Kosmologia

Viime viikolla oli merkittävä vuotuinen kosmologiakonferenssi COSMO. Tällä kertaa se järjestettiin Yhdysvaltojen Illinois’n yliopistossa ja pidettiin etänä. COSMO-konferenssien sarja alkoi vuonna 1997, tavoitteena tuoda hiukkasfyysikkoja ja kosmologeja lähemmäs. Nykyään kentät lomittuvat niin sujuvasti, että nuorempi sukupolvi ei edes tiedä, että niiden välillä oli aiemmin paljon epäilyä ja väärinkäsityksiä.

COSMO-konferenssien johtokunnan jäsen Leszek Roszkowski mainitsi hiukkasfysiikasta Nobelin saaneen Martinus Veltmanin ykskantaan todenneen, että kosmologia ei ole tiedettä. Mielipide meni Veltmanin mukana tammikuussa hautaan, ja nykyään kukaan fyysikko tuskin esittää tällaisia kommentteja. Hiukkasfyysikoiden mielenmuutosta on edesauttanut se, että kosmologiassa on tehty läpimurtoja havaintojen saralla, merkittävimpänä vuonna 1998 julkistetut (ja 2011 Nobelilla palkitut) maailmankaikkeuden kiihtyvästä laajenemisesta kertovat supernovahavainnot.

Tämän vuoden COSMOssa näkyi, miten havaintovetoista kosmologia on, ja miten paljon teknologia ja data-analyysi ovat kehittyneet. Kun vuonna 2000 aloitin tutkimuksen tekemisen, muotiterminä oli ”täsmäkosmologia”. Tässä COSMOssa ei sanaa juuri kuultu, koska havaintojen ja analyysin täsmällisyys on ilmeistä ja tunnustettua.

Kosmologiakonferensseissa ei julkisteta uusia tuloksia eikä artikkeleita, jotka ovat kaikki luettavissa nettiarkistosta arXiv, mutta ne tarjoavat kuratoidun läpileikkauksen alan tilanteeseen ja ponnistuslaudan ajattelulle. Ohjelma koostuu kutsutuista puheista, jotka ovat enimmäkseen katsauksia johonkin osa-alueeseen, ja osallistujien omasta työstään tarjoamista lyhyemmistä esityksistä. Vuosien varrella naisten osuus on kasvanut merkittävästi: tämän vuoden 24 kutsutusta puhujasta 15 oli (nimen ja ulkonäön perusteella oletetusti) naisia.

Gravitaatioaalloilla oli ansaitusti iso rooli. Uusien havaintojen (ja Nobelin) edesauttamana alan tutkimus on laajentunut nopeasti. Kun kosmisen mikroaaltotaustan epätasaisuuksien ensimmäiset mittaukset julkistettiin vuonna 1992, data-analyysin ja havaintojen ohella kehitettiin rivakasti erilaisia tapoja hyödyntää dataa ja lukea siitä kaikenlaista. Gravitaatioaallot ovat nyt samalla tavalla uusi väline, jolla voi luodata tähtien kehitystä, raskaiden alkuaineiden muodostumista, neutronitähtien rakennetta, värivuorovaikutusta, mustien aukkojen muodostumista, maailmankaikkeuden laajenemisnopeutta, yleistä suhteellisuusteoriaa, Higgsin kentän olomuodon muutosta ja niin edelleen. Kuten LIGO-koeryhmän Daniel Holz asian ilmaisi, tutkijat ovat odottaneet gravitaatioaaltojen datatulvaa, ja nyt se on alkanut.

Teoreetikot vaihtavat kentältä toiselle onnistumisten vetämänä ja epäonnistumisten hylkimänä. Monet vaikkapa gravitaatioaaltoihin tai Higgsin kenttään liittyvistä viime vuosien uusista ideoista olisi voitu keksiä vuosia sitten, mutta niitä ei tullut ajatelleeksi kuin havaintojen ja uuden keskustelun myötä. Teknologia ajaa havaintoja, mikä ruokkii teoriaa, mikä osoittaa miten analysoida ja tehdä havaintoja.

Teknologian kehitys näkyy myös teoriapuolella esimerkiksi siinä, että yhä enemmän tehdään raskaita yleisen suhteellisuusteorian kaikki yksityiskohdat huomioon ottavia numeerisia laskuja ei vain mustien aukkojen törmäyksistä, vaan myös kosmisesta inflaatiosta ja sen loppumisesta.

COSMOssa näkyi se, miten teoreetikot ovat siirtyneet enemmän lähellä havaintoja oleviin rajattuihin malleihin suureellisten rakennelmien sijaan. Säieteoria oli kuitenkin vielä vahvasti ainakin hengessä mukana. Tähän saattaa vaikuttaa se, että teoreettinen tutkimus on Yhdysvalloissa enemmän sidottua siihen, sen muotivirtauksiin ja persoonallisuuksiin kuin Euroopassa tai Aasiassa.

Gravitaatioaaltohavaintojen inspiroimana mahdollisten muinaisten mustien aukkojen tutkimus on paisunut. Pimeän aineen tutkimusta on päinvastoin ajanut havaintojen puute: koska aiemmin suosituinta ehdokasta pimeäksi aineeksi, nynnyä, ei ole löytynyt, tutkitaan yhä enemmän kaikenlaisia mahdollisuuksia.

Yksi esille nostettu idea on makroskooppinen pimeä aine, eli pimeä aine, joka koostuu isoista kasoista yhteen sitoutuneita hiukkasia, tavallisen aineen tapaan. Tällaisia kasoja pitäisi etsiä aivan eri tavalla kuin yksittäisiä hiukkasia, esimerkiksi katsomalla iskeytyykö tähtiin asteroidin massaisia mutta paljon nopeampia möykkyjä.

On aina havaintoja, jotka ovat ristiriidassa teorian kanssa, ja voi kestää kauan, ennen kuin saadaan selville, onko ongelma havainnoissa, teoriassa vai jossain oletuksissa. Yksi tällainen ongelma on se, miksi maailmankaikkeudessa näyttää olevan vain noin neljännes siitä määrästä litium-7:aa (eli ytimiä, joissa on kolme protonia ja neljä neutronia) mikä ennustetaan syntyvän maailmankaikkeuden ensimmäisten minuuttien aikana. Poikkeama on tilastollisesti erittäin merkittävä. Nyt on viimein paikallistettu uskottava mahdollinen selitys. Litiumin määrä on arvioitu vanhojen tähtien pinnalta, ja uusien havaintojen nojalla niissä on tapahtunut odotettua enemmän kehitystä, missä pinta on voinut sekoittua alempien kerrosten kanssa, johtaen litiumin tuhoon.

Kosmologian puhutuin ristiriita tällä hetkellä on se, että eri havainnot näyttävät antavat eri arvon maailmankaikkeuden laajenemisnopeudelle. Vastakkain ovat asettuneet erityisesti kosminen mikroaaltotausta ja kosmisen naapuruston supernovat. Kosmisen mikroaaltotaustan fysiikka tunnetaan erinomaisesti ja mittaukset ovat tarkkoja, mutta laajenemisnopeuden lukeminen siitä vaatii oletuksia pimeästä energiasta. Etäisyyksien mittaaminen supernovalla kärsii epävarmuuksista tähtien ja muiden järjestelmien mallintamisessa, mutta ei juuri riipu siitä, millaista pimeä energia on (tai onko sitä).

Havaintoryhmä Dark Energy Surveyn uudet havainnot galaksien jakaumasta ja gravitaatiolinssivääristymistä tukevat kosmisen mikroaaltotaustan tuloksia. Supernovapuolella on avautunut sisäinen ristiriita eri etäisyysmääritysten välillä: yksi tapa sopii yhteen kosmisen mikroaaltotaustan kanssa, toinen ei ja kolmas on liian epätarkka, että voisi vielä sanoa. Kovasta syynäämisestä huolimatta kummastakaan tarkassa supernovamenetelmässä ei ole löydetty virhettä. Mutta nyt tiedetään paremmin mistä katsoa, ja voi olla että huolellisen analyysin jälkeen osoittautuu että ongelmana on supernovien ympäristön tai tähtien mallintaminen, tai jokin muu kosmologian kannalta arkinen lähipiirin touhu.

COSMOn sävy oli innostunut ja eteenpäin ravaava. On uusia tarkkoja havaintoja, tulevia innostavia projekteja ja hauskoja teoreettisia ideoita. On silti syytä muista, että vaikka havainnot voivat tarjota vastauksen isoihin kysymyksiin –pimeän aineen luonne, kiihtyvän laajenemisen syy, inflaation yksityiskohdat, aineen ja antiaineen epäsuhdan synty– ja paljastaa jotain yllättävää, siitä ei ole mitään taetta. Kosmologian tulevaisuus on havaintojen arpapeliä.

16 kommenttia “Kuratoitu läpileikkaus”

  1. Eusa sanoo:

    Entäpä jos pimeän aineen möykyt ovatkin valovuosien kokoisia sumeita rakenteita?

    1. Syksy Räsänen sanoo:

      Jos pimeä aine koostuu hiukkasista, joiden massa on hyvin pieni, niiden aallonpituus voi tosiaan olla tähtitieteen mittakaavassa. Tällöin ei kuitenkaan ole kyse hiukkasten sidotuista tiloista (möykyistä), vaan yksittäisistä hiukkasista.

      Aiheesta lisää täällä:

      https://www.ursa.fi/blogi/kosmokseen-kirjoitettua/totalitaristinen-periaate-ja-vanhan-ajan-romantiikka/

  2. Sunnuntaikosmologi sanoo:

    Ketä voi kutsua ensimmäiseksi kosmologiksi, nykytieteen mielessä ? Albert Einstein ?
    Voinee kai sanoa että meni monta vuosikymmentä niin että kosmologia oli kokonaan teoreetikoiden juttu ?
    Tähtitieteilijäthän jakaantuvat teoreetikoihin ja havaitsijoihin, joista jälkimmäiset siis vastaavat fysiikan kokeilijoita.
    Ja nykyäänhän on sitten simuloijia jotka lasketaan teoreetikoiksi tai sitten omaksi ryhmäkseen.
    Onko nykyään olemassa sellainen identiteetti kuin havainnoitsija-kosmologi ?

    1. Syksy Räsänen sanoo:

      Kosmologia omana tieteenalanaan muodostui vasta pitkälle toisen maailmansodan jälkeen. Sitä ennen oli erilaisia fyysikoita, jotka tutkivat kosmologiaan liittyviä kysymyksiä muun ohella. En osaa sanoa, kuka olisi ensimmäinen kosmologi.

      Nykykosmologian voi sanoa syntyneen yleisen suhteellisuusteorian myötä, ensimmäisen siihen perustuvan kosmologisen mallin esitti Einstein vuonna 1917, ja kenttä kehittyi havaintojen myötä voimakkaasti 1920-luvulla.

      Simulaatioita tekevät sekä havaitsijat että teoreetikot, simulaatioiden tekijät eivät ole oma ryhmänsä.

      Havainnoiva kosmologia (observational cosmology) on tosiaan oma osa-alueensa.

  3. Martti V sanoo:

    Pimeän aineen hiukkasten detektoinnissa taidetaan olla hieman epätoivoisia. Onko muunnellut gravitaatioteoriat nostaneet profiilia viime aikoina?

    1. Syksy Räsänen sanoo:

      Itse asiassa pimeän aineen etsimisen suhteen ei yleisesti olla epätoivoisia (toisin kuin mitä tulee uusien hiukkasten löytämiseen kiihdyttimissä), koska ehdokkaita on niin paljon.

      Muokatut gravitaatioteoriat pimeän aineen kilpailijoina jatkavat pienellä liekillä. Tuki niille ei ole kasvanut, varmaankin siksi että 1) ei ole yhtäkään muokattua gravitaatioteoriaa joka selittäisi kaikki samat havainnot kuin pimeä aine, ja 2) pimeän aineen ennustukset ovat osuneet hyvin oikeaan.

      1. Martti V sanoo:

        Viime vuonna XENONin mahdollisesti detektoima aksioni olisi hyvin hedelmällinen löytö. Onko tarkempia eksperimenttejä tiedossa lähivuosina?

        1. Syksy Räsänen sanoo:

          Tosiaan, siitähän kirjoitin täällä: https://www.ursa.fi/blogi/kosmokseen-kirjoitettua/tonni-tankissa

          XENON-ryhmän kokeelta XENONnT odotetaan uusia tuloksia tänä vuonna (tai ehkä ensi vuonna). En tiedä etsivätkö muut tutkimusryhmät samanmassaista aksionia, kaikenlaista erilaisten aksionien metsästystä on kyllä käynnissä.

  4. Jyri T. sanoo:

    Onko pohdittu sellaista vaihtoehtoa, että jostain syystä pimeä aine ei pysty ”lokalisoitumaan” — eli jostain syystä tavallisten hiukkasten kyky muuttua tarvittaessa ”aallosta hiukkaseksi” ei koskekaan pimeää ainetta vaan se pysyy (nykyenergioissa) aina aaltomuodossa?

    Siinä tapauksessa pimeän aineen hiukkasia etsittäisiin aivan turhaan.

    1. Syksy Räsänen sanoo:

      Hiukkaset eivät muutu aallosta hiukkaseksi tai toisinpäin, vaan aalto ja hiukkanen ovat malleja, jotka kuvaavat joissakin olosuhteissa hiukkasten käytöstä. Ks. https://www.ursa.fi/blogi/kosmokseen-kirjoitettua/pikkuhyrrien-kertomaa/

      1. Jyri T. sanoo:

        Ymmärsit kysymykseni konkreettisemmin kuin oli minun tarkoitukseni. Kokeillaan uudelleen vähän abstraktimmalla otteella:

        Onko pohdittu malleja, joissa pimeän aineen aaltofunktio ei toimi samalla tavalla kuin tavallisen aineen aaltofunktio? Siten se ei vuorovaikuttaisi edes teoriassa millään muulla tavalla kuin gravitaation kautta ja sitä olisi mahdotonta havaita hiukkasilmaisimilla?

        Esimerkiksi fermionit ja bosonit käyttäytyvät (osin) eri tavalla koska spin. Ehkä tavallisella ainehiukkasella ja pimeällä ainehiukkasella on jokin kvantittunut opminaisuus, joka estää niitä reagoimasta toisiinsa?

        1. Syksy Räsänen sanoo:

          Malleja, joissa pimeä aine vuorovaikuttaa vain gravitaation (ja mahdollisesti muiden yhtä heikkojen) vuorovaikutusten kautta on tutkittu paljon. Tunnetuin esimerkki on supergravitaatioteorioiden gravitiino (gravitonin supersymmetrinen partneri).

  5. Antti sanoo:

    taas kerran mielenkiintoinen läpileikkaus kuukauden aiheesta!

    Riittääkö kvantti-painoteorian löytäminen selittämään galaksien
    rotaation teoreettisen virheellisyyden ja siten sulkisi pois
    pimeän aineen olemassaolon tarpeellisuuden tai vähintään muuttaisi asian
    hahmotusta tiedeyhteisöissä merkittävästi?

    Entä voisiko uusimmalla muon- g -2 löydöllä (mikäli se osoittautuu oikeaksi)
    mitään tekemistä pimeän aineen pois rajaamisessa?

    1. Syksy Räsänen sanoo:

      Kvanttigravitaatiolla ei ole mitään ilmeistä yhteyttä niihin havaintoihin, joita pimeä aine selittää.

      On malleja, joissa myonin magneettisen momentin poikkeama liittyy pimeään aineeseen, mutta useimmissa malleissa näin ei ole.

  6. Leo Sell sanoo:

    Litium probleemiin liittyen. Mainitsemasi ”litiumin tuho alaspäin hajoamisena” vanhoissa tähdissä on varmaankin se todennäköisin selitys. Eikö myös ole mahdollista, että osa puuttuvasta litiumista löytyy niistä galaksiytimien muustien aukkojen sylkemistä miljoonaasteisista purkaussuihkujen seittirihmastoista, galaksien välitiloissa, joissa vetykin on ionisoitunut näkymätttömäksi protoniainekseksi. Aikoinaan v. 2018 ilmoitettiin, muistaakseni 20.6:tta, että baryonistakin ”kateissa” ollutta ainetta oli löytynyt se puuttuva kolmannes juuri näistä seiteistä, Tässä Fabrizio Nicastron tukimusryhmässä havainnot paikallistettiin mukana olevien happiatomien perusteella, niissäoli yhä tallella kahdeksasta elektronistaan 2kpl, jotka sitten splittaamalla ytimiensä kanssa antoivat puuttuvat signaalit massojen arvioinneille.

    Eikö osa puuttuvaksi arvioidusta BBN – alun nukleosynteesi ainesosuuksista litiumin osalta voisi selittyä näissä korkeissa lämpötioissa näkymättömäksi, eli elektroniensa kanssa splittaamattomaksi, ionisoituneella litiummassalla?

    Artikkeli: ”Last of universes missing ordinary matter”/ astrophysics, Fabrizio Nicastro

    1. Syksy Räsänen sanoo:

      Mainitsemillasi rakenteilla ei ole mitään tekemistä mustien aukkojen kanssa.

      Litium-mittauksissa on kyse sen pitoisuudesta vanhoissa tähdissä. Ajatuksena on ollut, että koska litiumia ei synny missään tunnetuissa prosesseissa, niin vanhojen tähtien syntyessä niiden litium-pitoisuus vastaa alkuperäistä pitoisuutta, koska sitä ei ole vielä ehtinyt tuhoutua. On vaikuttanut siltä, että nämä tähdet eivät myöskään itse tuhoa litiumia, mutta nyt vaikuttaa siltä, että niin kuitenkin saattaa tapahtua.

      Tällä ei ole mainitsemiesi rakenteiden kanssa mitään tekemistä.

Vastaa

Sähköpostiosoitettasi ei julkaista. Pakolliset kentät on merkitty *


Hienosta karkeaan

29.6.2021 klo 18.41, kirjoittaja
Kategoriat: Kosmokseen kirjoitettua

Joskus törmää käsitykseen, että fysiikka on täsmällistä kuten matematiikka; jotkut puhuvat ”eksakteista tieteistä”. On kuitenkin erilaisia täsmällisyyden asteita, ja fyysikot ja matemaatikot suhtautuvat täsmällisyyteen eri tavoin.

Tutkimuksessaan Beamtimes and Lifetimes antropologi Sharon Traweek mainitsee, että teoreettiset hiukkasfyysikot arvostelevat kollegoitaan siitä, että nämä ovat liian matemaattisia. Pitääkin paikkansa, että teoreetikot yleensä pitävät sellaista matematiikkaa, joka on kehittyneempää kuin mitä he itse käyttävät joko tarpeettomana hienosteluna tai fysiikalle lähtökohtaisesti merkityksettömänä. Vastaavasti matematiikkaa, joka on vähemmän kehittynyttä kuin omassa käytössä oleva pidetään liian alkeellisena. Yksi hiukkasfysiikan puolelta tullut kosmologikollega kerran kutsui astrofyysikkojen käyttämää matematiikkaa kivikautiseksi. Vähemmän kehittyneiden matemaattisten menetelmien voidaan myös katsoa kertovan siitä, että niiden käyttäjä ei ole kovin terävä.

Tällä on pitkät perinteet: vielä suppean suhteellisuusteorian löytämisen aikoihin Albert Einstein piti kehittynyttä matematiikkaa tarpeettomana ja ajatteli, että fysikaalinen intuitio riittää. Tämä olikin kenties syynä siihen, että keskeisen oivalluksen siitä, että teoriassa ei ole kyse vain suhteellisesta ajasta ja avaruudesta, vaan absoluuttisesta aika-avaruudesta, teki Hermann Minkowski eikä Einstein. Yleisen suhteellisuusteorian kohdalla tämä lähestymistapa ei enää riittänyt, teoria vaati perehtymistä fysiikalle ennennäkemättömään avaruuksien hienosyiseen matematiikkaan, jonka matemaatikot olivat 1800-luvulla löytäneet.

Matemaattinen fyysikko Peter Goddard on maininnut vuodelta 1971 yhden kuvaavan esimerkin tuon ajan hiukkasfyysikoiden asenteesta (50 vuodessa on tapahtunut muutosta). Luennoitsija teki taitavasti monimutkaisia laskuja ja kääntyi yleisöön sanoen: ”Minulle on kerrottu, että tämän kaiken voi tehdä helpommin, jos käyttää ryhmäteoriaa. Mutta minä sanon teille, että jos olet vahva, niin et tarvitse ryhmäteoriaa.”

Traweek toteaa, että matemaatikot (kuten myös taiteilijat) ovat kuitenkin hiukkasfyysikoiden normaalin arvojärjestyksen ulkopuolella. (Sen huipulla on luonnollisesti hiukkasfysiikka, vaikka usko omaan erinomaisuuteen onkin Traweekin kartoittamasta 1970-80-luvusta horjunut, koska suosituista teorioista ei ole löytynyt hiukkaskiihdyttimissä merkkiäkään.) Matemaatikoiden täsmällisyyttä ja matemaattisten rakenteiden tuntemusta kunnioitetaan, vaikka siihen voidaan samalla suhtautua huvittuneesti, koska joitakin matemaatikkojen tutkimia asioita pidetään ilmeisinä tai yhdentekevinä. Tämä saattaa liittyä osittain siihen, että fyysikot eivät täysin hahmota, mitä matemaatikot oikein tekevät.

Minullakin oli aikoinaan sellainen hämärä käsitys, että matemaatikkojen suhde matematiikkaan on samanlainen kuin fyysikoilla, vain huolellisempi. Matematiikassa on kuitenkin tutkimusaiheena matematiikka, eli (fyysikon karkeasti sanomana) mahdolliset loogiset suhteet asioiden välillä. Fysiikassa taas matematiikka on työkalu luonnon ymmärtämiseen. Liiallista mielenkiintoa työkaluun työstettävän asian sijaan pidetään turhanaikaisena. Fysiikassa on täsmällisyyden asteita: teorioiden soveltamisessa matematiikkaa saatetaan kohdella reseptikirjana, vailla mielenkiintoa siihen miksi asiat pitävät paikkansa, toisaalta teorioiden muotoilussa voidaan mennä syvälle matemaattiseen rakenteeseen.

Täsmällisemmästä päästä ovat fyysikoiden ja matemaatikoiden rajalla olevat niin kutsutut matemaattiset fyysikot. Fyysikot pitävätkin heitä matemaatikkoina – en ole varma, mitä mieltä matemaatikot ovat. Matemaattiset fyysikot tutkivat matemaattisia rakenteita, joita pidetään fysiikalle oleellisina. Fyysikot eivät tosin aina ole niiden oleellisuudesta samaa mieltä. Olen kuullut erottelun, jonka mukaan fyysikot etsivät menetelmiä ongelmiensa ratkaisemiseen, kun taas matemaattiset fyysikot etsivät ongelmia, joihin käyttää menetelmiään. (Puhuja ei ollut matemaattinen fyysikko.)

Fields-mitalilla palkittu matemaatikko Alain Connes on todennut, että (kenties matemaattisten) fyysikoiden ”jokseenkin vapaa” suhtautuminen matematiikkaan on ”omaperäinen ja tuottoisa”, ja sillä on ollut hyvin positiivinen vaikutus. Hän viittasi erityisesti säieteorian yhteydessä ja liepeillä tehtyyn tutkimukseen, missä fyysikot ovat löytäneet uusia matemaattisia rakenteita ja yhteyksiä, joita matemaatikot ovat sitten tarkemmin ymmärtäneet. Säieteoria lieneekin toistaiseksi antanut matematiikalle enemmän kuin fysiikalle.

Koska fyysikot ovat vähemmän täsmällisiä ja käyttävät rouheampaan ajatteluun soveltuvia työkaluja, he pystyvät etenemään matemaatikkoja nopeammin. Kolikon toinen puoli on se, että toisin kuin matemaatikoilla, fyysikoilla on usein epämääräinen kuva siitä, mitä tarkalleen on todistettu ja mitä ei, ja mitä oletuksia jokin tulos edellyttää: lyhyesti sanottuna, mikä on totta. Fysiikassa onkin niin kutsuttuja folkloreteoreemoja, kansanperinnelauseita, joissa on epäselvää mistä tuloksessa on tarkalleen kyse, mitä oletuksia oikein tehdään, ja joita kukaan ei ole ehkä koskaan osoittanut todeksi. Usein ne pitävät tarpeeksi hyvin paikkansa, joskus kompastuu niiden puutteisiin – ja toisinaan tapahtuu edistystä, kun vedenpitäväksi luullusta asiasta paljastuukin rako, josta avautuu uusi näkymä.

23 kommenttia “Hienosta karkeaan”

  1. Cargo sanoo:

    ”Minulle on kerrottu, että tämän kaiken voi tehdä helpommin, jos käyttää ryhmäteoriaa. Mutta minä sanon teille, että jos olet vahva, niin et tarvitse ryhmäteoriaa.”

    Kyseessä lienee hiukkasten kulmaliikemäärien tai spinien yhteenlasku sekä Clebsch-Gordan-kaavan soveltaminen? (Itse en ole aiheeseen tutustunut.)

    ”Matemaattiset fyysikot tutkivat matemaattisia rakenteita, joita pidetään fysiikalle oleellisina. Fyysikot eivät tosin aina ole niiden oleellisuudesta samaa mieltä.”

    Eräs melko abstraktin matematiikan tulos on se, että neliöintegtoituvien funktioiden avaruudessa itseadjungoituvien operaattorien negatiiviset ominaisarvot muodostavat diskreetin joukon. Fysikaalinen vastine on luonnollisesti vetyatomin hamiltoni sekä sidotut ja siten negatiiviset energiatilat (vedyn spektri). Myös arkiajattelulle vieraan epätarkkuusperiaatteen esittäminen Fourier-muunnoksen avulla on suht yllättävä tulos.

    1. Syksy Räsänen sanoo:

      Ei ollut. Ryhmäteoriaa käytetään fysiikassa useissa yhteyksissä. Lisätietoja tässä tapauksessa on tekstissä linkatussa blogimerkinnässä https://www.math.columbia.edu/~woit/wordpress/?p=657

  2. Erkki Kolehmainen sanoo:

    Nathan Seiberg pohdiskelee fysiikan ja matematiikan yhteyttä Quantum Magazinen haastattelussa.

    https://www.quantamagazine.org/nathan-seiberg-on-how-math-might-reveal-quantum-gravity-20210624/

  3. Joksa sanoo:

    YST:n laatija suhtautui itsekin suurella varauksella joihinkin matemaattisiin ennusteisiin. Matematiikassahan hypoteesin johtaessa ristiriitaan hypoteesi hyljätään mutta kosmologiassa ei koska useat muut teorian ennusteet ovat olleet oikeita.

    YST:n aikakuvaushan on perusristiriitainen: kaikki mahdolliset havainnoijat elävät samaa yhteistä nyt-hetkeä maailmankaikkeuden iän suhteen mutta ominaisaikamatemaatikot eivät sitä ole onnistuneet löytämään. Löytyisiköhän jos korjataan havainnoijien ajat gravitaation ja liikkeen vaikutuksesta taustasäteilyn nolladipolaariin ja merkataan yhtäsuuriksi?)

    1. Syksy Räsänen sanoo:

      Hypoteesien ja havaintojen suhteesta fysiikassa, ks. https://www.ursa.fi/blogi/kosmokseen-kirjoitettua/edistys-ja-rappio/

      Kosmologiassa (eikä missään muuallakaan) ei ole mitään kiistattomia yleisen suhteellisuusteorian vastaisia havaintoja. Päin vastoin, yleinen suhteellisuusteoria on tarkasti ja onnistuneesti ennustanut lukuisia havaintoja yli sata vuotta.

      Kuvauksesi yleisen suhteellisuusteorian aikakäsityksestä ei pidä paikkaansa, eikä yleisessä suhteellisuusteoriassa ole sisäisiä ristiriitoja. (Paitsi singulariteetit, jotka rajaavat teorian pätevyysaluetta.) Ajasta yleisessä suhteellisuusteoriassa vähän täällä: https://www.ursa.fi/blogi/kosmokseen-kirjoitettua/kaksi-tarinaa-ajasta/

      Tämä riittäköön tästä.

  4. Kari Leppälä sanoo:

    Luin mielenkiinnolla, koska aavistan että tässä ollaan jollain lailla luonnontieeen perusasioiden äärellä. Omat taitoni eivät valitettavasti riitä keskustelun kunnolliseen seuraamisen mutta kiinnostavaa joka tapakseessa. Filosofit puuhaavat mielellään tieteen ”raja-alueilla”, vakka jopa perusasioiden tajuaminen vaihtelee kovasti. Et maininnut filosofeja. Joko he ovat pudonneet kelkasta?

    1. Syksy Räsänen sanoo:

      Mitä tarkoitat kelkasta putoamisella? Filosofit käyttävät matematiikasta lähinnä vain logiikkaa (ja sitäkin vain pieni osa), ja ymmärtääkseni näillä tutkimuksilla on vain jonkin verran annettavaa matemaattisille loogikoille, eikä mitään muille matemaatikoille saati fyysikoille.

      1. Eusa sanoo:

        Matematiikassa ja matemaattisessa fysiikassa on runsaastikin aihetta filosofialle; esim. rakenteellisen emergenssin ja kätisyyden kysymykset sekä mittausongelmassa useat näkökulmat.

        Kuitenkin filosofia on suuntautunut laajalti mm. ontologisiin erikoiskysymyksiinsä ja on harvoja matematiikkaa ja fysiikkaa syvällisesti opiskelleita filosofeja. Toisaalta matematiikan/fysiikan alan tutkijoiden lipsahtelu filosofointiin ei ole muodissa nyt.

        1. Syksy Räsänen sanoo:

          Päin vastoin, kosmologisiin kysymyksiin ja säieteoriaan liittyen fyysikot ovat viime aikoinan kirjoitelleet paljon tieteen filosofiasta, ja tieteen filosofit ovat käsitelleet paljon näitä kysymyksiä.

          1. Eusa sanoo:

            Ok. Tietysti filosofiinen näköala on maastotilannesidonnainen. Säieteorian merkityskysymykset ovat mielenkiintoisia mutta ehkä tarvitaan sieltä vielä niitä ennustevoimaisuuksia.

            Tarkoitan lähinnä fundamentaalimman jo todennetun matemaattisen fysiikan filosofista edistämistä. Tulkintojen seoksesta voisi saada irti muutakin kuin inttämistä, osin denialististakin…

            Ontologisen arvioinnin keskeneräisyys mielestäni näyttäytyy sellaisissa kysymyksissä kuin ajassa taaksepäin matkustamisen mahdollisuuden elättely – tuntuisi, että fysiikan, matematiikan ja filosofian yhdistelmällä moisen mahdottomuuden osoittaminen pitäisi olla 99% varmuudella selviö.

            Lisäksi luonnonfilosofisten käsitteiden konsensusmäärittelyissä olisi paljon työsarkaa. Täsmällinen sitominen ilmiöihin ilman tulkintoja tulisi olla kirkas tavoite.

          2. Syksy Räsänen sanoo:

            Se, onko ajassa taaksepäin matkustaminen mahdollista tai mahdotonta ei ole filosofinen tai matemaattinen kysymys, vaan se riippuu siitä, millaisia fysiikan lait ovat. Tällä hetkellä tuntemuksemme niistä ei riitä kertomaan mikä on asian laita, eikä mikään määrä filosofisia tai matemaattisia tarkastelua muuta asiaa.

          3. Syksy Räsänen sanoo:

            Aikamatkustus menee sen verta kauas merkinnän aiheesta, että ei siitä sen enempää.

      2. Kari Leppälä sanoo:

        No tarkoitin että vieläkö ne jaksaa olla kiinnostuneita fysiikasta. Oma kiinnostukseni kohdistuu ”tekoälyyn”, laitoin tosin lainausmerkkehin. Aihe sivuaa kognitiota, itsen käsitettä ja kokemista; en näe niissä mitään erityisen outoa mutta ns mielenfilosofit on sillä puolella aktiivisia. Logiikkaa tosiaan näkee joskus, mutta metodi näyttäisi olevan käsiteanalyysi ja omien kieleen nojaavien ajatusrakennelmien rjkentelu. Sellainen filosofi kuin Alfed Mele lähtee tästä ja päätyy kumoamaan kaikki luonnontieteet, jollain fundamentaalilla tavalla väärinä. No ehkä hän on ääriajattelija.

        1. Syksy Räsänen sanoo:

          En osaa antaa kunnollista vastausta, koska en juuri tunne filosofiaa. Mutta kuten toiseen kommenttiin vastasin, viime vuosina ovat filosofit kommentoineet ahkerasti kosmologiaan, säieteoriaan ja teorioiden varmentamiseen liittyen. Fysiikan kannalta siltä on ollut vain marginaalisesti merkitystä.

        2. Kari Leppälä sanoo:

          Nimi tietysti oikein kirjoitettuna Alfred Mele …

  5. Lentotaidoton sanoo:

    Räsänen: ”Se, onko ajassa taaksepäin matkustaminen mahdollista tai mahdotonta ei ole filosofinen tai matemaattinen kysymys, vaan se riippuu siitä, millaisia fysiikan lait ovat. Tällä hetkellä tuntemuksemme niistä ei riitä kertomaan mikä on asian laita, eikä mikään määrä filosofisia tai matemaattisia tarkastelua muuta asiaa.”

    Napakasti sanottu. Tosiaan maailma näyttää olevan pullollaan (kymmeniä/satoja) enemmän tai vähemmän perusteltuja näkemyksiä ensinnäkin siitä, mitä voidaan olettaa jo tunnettujen fysiikan lakien perusteella tai etenkin niiden ”tunnettujen ei tunnettujen” tai ”ei tunnettujen ei tunnettujen” lakien perusteella; eli liikkuen reilusti sekä suhteellisuusteorian että tunnetun kvanttifysiikan ulkopuolella (tai ainakin ulkopuolella niiden ”normitulkinnan”).

    Usein näitä lukiessa tuntuu siltä, että eritasoisia filosofisia ajatelmia ei näemmä (turhaan) rajoita niin triviaali periaatteellinen asia kuin fysiikan lait (tai niitä ”sävelletään” surutta opportunistisesti oman rakennelman ylläpitoon). Sanottakoon kuitenkin, että tietysti meillä on paljon perin vakavasti otettaviakin fysiikan rakennelmia yli nyt tunnetun fysiikan, ei siis pelkkää filosofista pohdintaa.

    1. Eusa sanoo:

      Säieteorian voi katsoa olevan matemaattisfilosofinen rakennelma, jota ”fysiikan lait” eivät rajoita – toki halua on löytää se versio, jossa lainalaisuudet totetuisivat…

      1. Syksy Räsänen sanoo:

        Ei säieteoriasta sinällään nyt sen enempää, elleivät kommentit koske matematiikan ja fysiikan suhdetta.

  6. Antti sanoo:

    Hei Syksy oletko perehtynyt tähän naisen töihin tai onko
    nimi tullut vastaan missään?

    ”seuraava einstein” kuullostaa aika lupaavalta
    naisella on täydet pisteet kolmessa eri parhaassa
    yliopistossa Usan maaperällä ja neiti on rakentanut toimivan lentokoneen
    14 vuotiaana vanhempien autotallissa.

    https://www.youtube.com/watch?v=jQhn7XFaBug&t=2146s

    1. Syksy Räsänen sanoo:

      En.

  7. Gc sanoo:

    Jotkut fyysikot tuntuvat halveksivan erityisesti aksiomaattisia teorioita. Esimerkiksi juuri edesmenneen Weinbergin uusimassa kirjassa tästä on merkkejä. Einstein kai aikanaan yritti sovittaa Maxwellin aksiooomat omaan teoriaansa, mutta käsitykseni on että paljastui että Maxwellin aksiiomat ovat lähes kaikki vain yleisia matemaattisia totuuksia, ja vain yhdellä niistä on matemaattinen sisältö. On myöhemmin osoitettu että nämä aksiiomat pelaavat hyvin yleisenkin suhteellisuus teorian kanssa. Aksiomaattinen teoria juuri opettaa sen mitä oletetaan ja mistä oikeasti voidaan päätellä mitä.
    Enkvistikin on minusta sanonut kummallisia asioita Tieteessä Tapahtuu lehdessä esimerkiksi siitä että kvanttifysiikka on joten ”ylimatemaattista.” Minä uskon että joku joskus osaa saada jopa polkuintegraalit matemaattisesti hyvinmääritellyiksi. Ei ole syytä olettaa että juuri nyt matematiikan ja fysikan polut ovat eronneet lopullisesti. Minusta monesti matematiikka menee näpertelyksi ilman teoreettista fysiikkaa. On monia rakenteita joita tutkitaan, mutta jotka ovat jossain mielessä keinotekoisia, siis minusta. Esimeksi Diracin delta-mitta tai delta-distribuutio on valtavan tärkeä matematiikassa. Nykyiset yritykset polkuintegraalien määrittelyksi distribuutioiden kautta ovat minusta turhan abstrakteja. Teoreettiset fyysikot tekevät suurelta osin matematiikan hyvin, enkä usko että distribuutiot ovat siellä kovin suosittuja. Esimerkiksi Lebesguen integraalia ei opeteta, mikä on aivan oikein. Lebesguen integraalia tarvitaan siihen että funktio avaruuksista tulee täydellisia normi(en) määräämien topologioiden suhteen, mutta niissä harvoissa tapauksissa kun on epäselvää onko joku raja-arvo olemassa puhutaan fysiikassa yleensä vain jostain tietyistä funktioista, joten koko Hilbertin avaruutta ei mielestäni tarvita (Myös Weyl kirosi Hilbertin avaruutta) . Mitkä tahansa kantakertoimet (a_i)_{i=1}^\infty, joille pätee \sum_{i=1} |a_i|^2 kyllä määrittävät yhdessä minkä tahansa kannan kanssa neliöintegroituvan funktion. Jo yleinen neliöintegroituva funktio on aika abstrakti tapaus, puhumattakaan distribuutiosta. Fyysikot tekevät aivan oikein – siitä on etua, jos funktioiden tai potentiaalien joukko on hallittavampi.

    1. Syksy Räsänen sanoo:

      Maxwellin sähkömagnetismin aksioomat eivät ole ”yleisiä matemaattisia totuuksia”, ja kaikilla niistä on ”matemaattinen sisältö”.

      Tämä riittäköön tästä.

      1. Gc sanoo:

        Tarkoitin että vain yhdellä niistä on varsinaisesti fysikaalinen sisältö. Kaikilla niillä on tietysti matemaattinen sisältö. Veikkaanpa että haluat vielä muuttaa vastaustasi, mutta se on vain minun veikkaukseni. Ja tarkoitin toki että mitkä tahansa kantakertoimet (a_i)_{i=1}^\infty, joille pätee \sum_{i=1} |a_i|^2 k < \infty minkä tahansa ortonormaalin kannan kanssa….

Vastaa

Sähköpostiosoitettasi ei julkaista. Pakolliset kentät on merkitty *

Sankarien riisumista

22.6.2021 klo 20.23, kirjoittaja
Kategoriat: Kosmokseen kirjoitettua

Tammikuisessa monimuotoisuuteen ja naisten asemaan fysiikassa keskittyvän NORNDiP-verkoston konferenssissa suositeltiin antropologi Sharon Traweekin teosta Beamtimes and Lifetimes: The World of High Energy Physics. Sain kirjan nyt luettua, ja se onkin mielenkiintoinen katsaus fyysikoihin.

Traweek tutki kokeellisten hiukkasfyysikkojen yhteisöä Yhdysvalloissa ja Japanissa 1970- ja 80-luvulla. Osa kirjan huomioista on vanhentuneita, mutta siinä on myös paljon oivalluksia, jotka pätevät yhä ja myös teoreetikoihin. Noihin aikoihin toisen maailmansodan jälkimainingeissa työnsä aloittanut fyysikoiden sukupolvi oli siirtymässä pois johtoasemista. Tämä fyysikoiden joukko suurelta osin määritti nykyisenkin kokeellisen toimintatavan ja kulttuurin.

Osallistuminen Yhdysvaltojen armeijan projekteihin sodan aikana oli opettanut fyysikoita hallinnoimaan suuria projekteja, viestimään päättäjien kanssa ja haalimaan rahoitusta. Toisaalta fysiikka oli osoittanut sodassa käyttökelpoisuutensa, mikä sai poliitikot alttiiksi rahoittamaan sitä avokätisesti. Kuten Traweek kuivasti huomauttaa, hiukkasfyysikot pitävät kuitenkin kohtuuttomana sitä, että suuri yleisö yhdistää heitä mitenkään yhdistetään aseteknologiaan, ja sodan jälkeen tehtiinkin rajanveto rauhanomaisen ja sotilaallisen tutkimuksen välillä.

Traweek kirjoittaa siitä sitä, miten tärkeää antropologille on sekä pitää ulkopuolinen näkökulma tutkimaansa yhteisöön että ymmärtää se sisältäpäin. Tämän ulkopuolisuuden saamiseksi hän muuttikin Yhdysvalloista Japaniin tutkimaan hiukkasfyysikoita myös siellä, erityisesti KEK-laboratoriossa. Vaikka Traweek yleistää turhan helposti japanilaisesta kulttuurista, hänen vertailussaan Yhdysvaltoihin on kiinnostavia havaintoja.

Traweek toteaa, että yksi keskeinen syy yhdysvaltalaisten laboratorioiden (erityisesti SLACin) japanilaisia parempaan menestykseen oli se, että Japanissa hallitus seurasi tarkkaan tutkimusprojektien rahankäytön yksityiskohtia, ja laitteita rakennettiin yritysten tukemisen ehdoilla. Yhdysvalloissa projekteilla oli enemmän vapautta käyttää rahaa ja enemmän valtaa rakentaa laitteet itse, ja ne tekivät yhteistyötä yritysten kanssa omista lähtökohdistaan.

Toisaalta vallan keskittyminen johtajalle ja sanomaton mutta tiukka hierarkia Yhdysvalloissa kummastutti japanilaisia tutkijoita. Traweekin mukaan Japanissa koko henkilöstö puhui laboratoriota koskevista kysymyksistä, ja johto teki päätöksensä keskusteltuaan asiasta laajasti. Yhdysvalloissa tärkeistä asioista eivät usein edes tienneet muut kuin johtoporras. Hän vertaa japanilaista yhteisöä perheeseen ja yhdysvaltalaista urheilutiimiin. Traweekin mukaan monet yhdysvaltalaiset tutkijat eivät edes tulleet ajatelleeksi, että yhteistyötä ja yhteistä päätöksentekoa korostava järjestely voisi tuottaa paremman tuloksen kuin keskinäinen kilpailu ja vahva johtaja.

Tähän liittyy uskomus siitä, että fysiikan yhteisössä korkein status on osoitus parhaista taidoista, ja palkkiot (kuten rahoitus ja kokeissa käyttöön annettu aika) jakautuvat aina ansioiden mukaan. Kuten Traweek kartoittaa, samaan aikaan fyysikot ovat usein sitä mieltä, että tietyt korkeassa asemassa olevat henkilöt (tyypillisesti kilpailevista ryhmistä) tekevät aivan onnettoman huonoa työtä. Tällaiset ristiriitaiset käsitykset ovat vieläkin osa hiukkasfysiikan yhteisöä.

Traweek setvii hyvin sitä, miten hatara käsitys fyysikoilla on alansa historiasta. Fyysikoita kiinnostaa menneisyydessä vain heidän suorat edeltäjänsä; vääriksi osoittautuneet ideat ja virheelliset kokeet eivät kuulu kertomukseen. Traweek ei juuri liioittele kirjoittaessaan, että ”heidän fysiikan historiansa on lyhyt hagiografia ja lista ihmeitä”. Tämä kuva rakentuu oppikirjoista alkaen, minkä yksi syy on se, että niiden tarkoituksena on opettaa tekemään fysiikan tutkimusta, ei kirjoittamaan fysiikan historiaa. Oppikirjat myös koulivat fyysikkojen yhteisöön ja opettavat millainen on hyvä fyysikko.

Traweekin kaikki analyysi ei vakuuta. Hän esimerkiksi kirjoittaa, että fysiikan kursseilla opetetaan vain ratkaisemaan tiettyjä ongelmia ja tunnistamaan miten uudet ongelmat ovat samanlaisia: siellä ei opeteta ” induktiota eikä deduktiota vaan analogista ajattelua”. Tämä analyysi vaikuttaa vieraalta, enkä oikein ymmärrä mitä hän tarkoittaa. Asiaan voi vaikuttaa se, että Traweekilla ei ole luonnontieteellistä koulutusta ja hän vaikuttaa perustavan johtopäätöksensä paljolti oppikirjoihin. Fysiikan oppikirjoja lukemalla niitä on varmasti vaikea ymmärtää, millaista on opiskelu, joka keskittyy ihmistieteitä enemmän tekemiseen eikä lukemiseen. Fysiikassa ei esimerkiksi opiskella erikseen fysiikan metodeja, vaan asioita opitaan käytännössä vähän kuin käsityössä sen mukaan mitä tarvitaan. Tämä suhde metodeiksi kutsuttuihin asioihin on erilainen kuin ihmistieteissä, ja niiden pääasiallinen oikeutus on se, että ne toimivat, mistä kokeet ja matematiikka antavat puolueettoman tuomion.

Traweekin mukaan antropologille on tärkeää olla ottamatta kantaa tutkimansa yhteisön uskomusten todenperäisyyteen, mikä on käsittääkseni tavallinen näkemys. Tämä tuntuu oudolta, koska uskomusten suhde todelliseen havaittuun maailmaan kertoo myös niiden syistä ja auttaa niiden ymmärtämistä. Uskomus siitä, että Aurinko nousee aamulla juontuu eri syistä kuin uskomus siitä, että Kristus nousee kuolleista.

Antropologinen näkökulma tuottaa kuitenkin valaisevia havaintoja esimerkiksi suhteessa säteilyturvallisuuteen. SLACissa kiihdytinalue on rajattu ulkopuolisesta julkisesta tilasta aidalle, minkä Traweek yhdistää kulttuuriseen rajanvetoon puhtaan ja saastuneen välillä, ja salatun ja kielletyn yhteyteen. Tätä voisi pitää ylitulkintana ja nähdä asian vain säteilyturvallisuuskysymyksenä, ellei ottaisi huomioon, että alueella ei SLACin mukaan oleellisesti ole säteilyvaaraa, eikä Japanin KEK-laboratoriossa ole vastaavia aitoja.

Muuten Traweekin kuvaukset kokeista ja laitteistoista eivät ole kirjan kiinnostavinta antia. Ne havainnollistavat kyllä sitä, miten paljon teknologia on kehittynyt. SLACin koeasema K-Zero rekisteröi 24 miljoonaa reaktiota viidessä ja puolessa vuodessa ja seuraavan sukupolven LASS rekisteröi saman määrän kolmessa kuukaudessa; nykyään LHC rekisteröi 40 miljoonaa törmäystä sekunnissa. Traweek kommentoi, että hiukkasfysiikka eroaa monista muista tutkimuksen aloista siinä, että laitteet edustavat kehittyneintä saatavilla olevaa teknologiaa, ja fyysikot itse suunnittelevat ja rakentavat ne itse, eivät hanki valmiina.

Kokeiden kuvauksen yhteydessä olevat Traweekin selitykset fysiikan sisällöstä menevät yllättävän usein hieman pieleen ottaen huomioon, miten pitkään hän oli tekemisissä fyysikoiden kanssa ja työskenteli sitä ennen SLACin suurelle yleisölle suunnatussa vierailijaohjelmassa. Myös laitteiden lukeminen sukupuolisten ja seksuaalisten vertauskuvien kautta on paikoitellen liioiteltua. Traweek esimerkiksi esittää SLACin (Stanford Linear Accelerator Center) esimerkkinä laboratorioiden ja kokeiden nimien sukuelimellisyydestä. (Englannin sana slack, joka äännetään samalla tavalla kuin SLAC, tarkoittaa velttoa.)

Enimmäkseen Traweekin huomiot sukupuolten jakaumasta ja niihin liittyvistä asenteista fyysikoiden keskuudessa ovat kuitenkin perusteltuja. Hän käsittelee myös jonkun verran rodullistettujen asemaa yhdysvaltalaisessa fysiikan yhteisössä, mutta tämä tarkastelu jää pinnallisemmaksi. Epätasa-arvo on yhä ongelma, mutta edistystä on tapahtunut, ja nykyään avoin naisten vähättely on harvinaisempaa. Minua yllätti, että 70-80-luvulla fyysikot pitivät itsestään selvänä, että he ovat naimisissa, ja juoruilivat ahkerasti naimattomien miesten vastuuttomasta romanttisesta elämästä tai seksielämästä, minkä katsottiin haittaavan heidän työtään. Tällaiseen en ole Euroopassa törmännyt, enkä usko sen elävän enää Yhdysvalloissakaan.

Traweekin havainnot juorujen eli epämuodollisen yhteisöä koskevan puheen tärkeästä roolista tiedon välittämisessä pitävät muuten monin osin vieläkin paikkansa. Samoin hänen kartoituksensa fyysikkojen omakuvan kehityksestä romanttisista sankareista (mikä opitaan oppikirjoista) kohti haavoittuvampaa ylevyyttä tuntuu yhä osuvalta. Huomiot aggressiivisesta keskinkertaisuuden väheksymisestä, käsityksestä fyysikoiden erinomaisuudesta verrattuna muihin tutkijoihin, ja tarpeeksi rajatuissa raameissa pysyvän itsenäisen ajattelun palkitsemisesta pätevät nekin vielä.

Traweek kirjoittaa liikuttavasti siitä, miten fyysikoiden pelot ja käsitys ajasta kehittyvät. ”Uransa aikana fyysikko oppii menneisyyden mitättömyyden, pelon siitä, että on liian vähän aikaa nykyhetkessä, ja ahdistuksen siitä, että tulee tarpeettomaksi liian nopeasti edistyvän tulevaisuuden myötä.” Traweekin mukaan fysiikan kulttuuri ”huolella ylläpitää näitä kauhuja, kuin ne olisivat hyvien fyysikoiden välttämättömiä moottoreita”. En ole varma, onko tämä totta, mutta aikaan liittyvät huolet ovat iso osa fyysikon arkea. Tämän tiimoilta kirjassa on yksi havainto, joka tuntuu nykynäkökulmasta surulliselta: kirjoitusaikaan vain korkeimmissa asemissa olevien fyysikoiden odotettiin hankkivan rahoitusta, nykyään se on kaikkien riesa.

Traweek aloittaa kirjansa siitä, miten fyysikot yleensä esitetään elämää suurempina sankareina, maailmankaikkeuden mysteerien papistona. Kirjan yksi tarkoitus on demytologisoida fyysikot, kuvata sitä, miten he todella ajattelevat, toimivat, puhuvat, ja ylläpitävät yhteisöään. Tarkastelun syvyyden vaatima rajoittuma kokeelliseen hiukkasfysiikkaan kahdessa maassa vie hieman pohjaa sen yleistyksiltä. Silti kirja auttoi paremmin jäsentämään omia kokemuksiani teoreettisessa hiukkasfysiikassa ja sille läheisessä kosmologiassa, vähentämättä romanttista kiintymystä niihin.

2 kommenttia “Sankarien riisumista”

  1. Cargo sanoo:

    ”Traweek kirjoittaa liikuttavasti siitä, miten fyysikoiden pelot ja käsitys ajasta kehittyvät. ’Uransa aikana fyysikko oppii menneisyyden mitättömyyden, pelon siitä, että on liian vähän aikaa nykyhetkessä, ja ahdistuksen siitä, että tulee tarpeettomaksi liian nopeasti edistyvän tulevaisuuden myötä.’ Traweekin mukaan fysiikan kulttuuri ’huolella ylläpitää näitä kauhuja, kuin ne olisivat hyvien fyysikoiden välttämättömiä moottoreita’.”

    Mutta eikö juuri nykyaikana ole erittäin todennäköistä, ettei fysiikan tutkija tee koko uransa aikana yhtä ainutta merkittävää löytöä? Kokeeko teoreettisen fysiikan supersuhdanne auringonlaskun ja saavatko fyysikot asennoitua olemaan maailmanselittäjinä samalla viivalla fenomenologien kanssa?

    ”Traweek aloittaa kirjansa siitä, miten fyysikot yleensä esitetään elämää suurempina sankareina, maailmankaikkeuden mysteerien papistona. Kirjan yksi tarkoitus on demytologisoida fyysikot, kuvata sitä, miten he todella ajattelevat, toimivat, puhuvat, ja ylläpitävät yhteisöään.”

    Mutta eikö vastapainoksi voisi sanoa, että vuorostaan humanistit kykenevät käsittämään asioita, jotka ovat luonnontieteilijöille täysin käsittämättömiä? 🙂

    Anyway, ainakin Einstein osasi ironisoida fysiikan sankarimyyttiä: ”When we are working at something, we come down from our high logical horse and sniff around with our nose to the ground. Then we obliterate our traces in order to become more God-like.”

    1. Syksy Räsänen sanoo:

      Hiukkasfysiikka on tietysti vain pieni osa fysiikkaa, ja fysiikassa kokonaisuutena tehdään koko ajan löytöjä.

Vastaa

Sähköpostiosoitettasi ei julkaista. Pakolliset kentät on merkitty *

Suti ja vasara

30.5.2021 klo 17.10, kirjoittaja
Kategoriat: Kosmokseen kirjoitettua

Hiukkaskiihdyttimissä voi löytää uusia hiukkasia kahdella tavalla. Suoraviivaisinta on iskeä kaksi hiukkasta toisiinsa niin kovaa, että törmäyksessä syntyy etsitty hiukkanen. Toisaalta vaikka kokeessa ei syntyisi uutta hiukkasta, sen olemassaolon voi päätellä siitä sen vaikutuksesta tunnettuihin hiukkasiin.

Jo ennen kuin Higgsin hiukkanen luotiin protonien törmäyksillä vuonna 2012, sen olemassaolosta oli paljon epäsuoraa todistusaineistoa. Higgs antaa massan heikkoa vuorovaikutusta välittäville W– ja Z-bosoneille ja vaikuttaa niihin liittyviin hiukkasreaktioihin. Olisi ollut vaikea selittää muiden hiukkasten käytöstä ilman Higgsiä.

Yksi tällä hetkellä kiinnostava väylä uusien hiukkasten epäsuoraan havaitsemiseen on B-mesonit. Tiistaina kokeellinen fyysikko Mitesh Patel Iso-Britannian Imperial Collegesta puhui Helsingin yliopiston Fysiikan tutkimuslaitoksen seminaarisarjassa niiden uusimmista kuulumisista.

B-mesonit ovat yhdistelmähiukkasia, jotka koostuvat yhdestä bottom-antikvarkista ja yhdestä up-, down-, charm- tai strange-kvarkista. Kvarkit ovat alkeishiukkasia, jotka esiintyvät vain sidottuina tiloina, joita kutsutaan hadroneiksi. (Paitsi hyvin kuumissa ja/tai tiheissä olosuhteissa varhaisessa maailmankaikkeudessa ja ehkä neutronitähtien keskustassa.) Hadroneita on pääasiassa kahdenlaisia: baryoneita, joissa on kolme kvarkkia ja mesoneita, joissa on yksi kvarkki ja antikvarkki. Myös monimutkaisempia yhdistelmiä on havaittu. Protoni on ainoa pysyvä hadroni. Vapaa neutroni hajoaa 15 minuutissa, muut hadronit hajoavat sekunnin murto-osassa.

Hiukkasten hajoaminen on sattumanvarainen tapahtuma, ja voidaan ennustaa vain se, millä todennäköisyydellä mitäkin hiukkasia syntyy ja mihin suuntaan ne liikkuvat. B-mesonin tapauksessa tämä voidaan periaatteessa laskea hiukkasfysiikan Standardimallista. Ongelmana on se, että kvarkit liikkuvat ja vuorovaikuttavat hadronien sisällä monimutkaisella tavalla. Niinpä on vaikea laskea tarkasti, miten mesoni hajoaa, etenkin silloin kun lopputulosten joukossa on toinen mesoni. Ongelma voidaan ohittaa siten, että ei verrata vain havaintoja ja teoriaa, vaan myös havaintoja toisiinsa.

Esimerkiksi ei osata tarkkaan laskea B-mesonin hajoamista K-mesoniksi ja elektroni-positroni-pariksi, eikä K-mesoniksi ja myoni-antimyoni-pariksi, eikä K-mesoniksi ja tau-antitau-pariksi. Mutta kaikissa kolmessa laskussa on sama ongelmakohta, eikä se vaikuta hajoamisten suhteeseen. Voidaan siis ennustaa, että kaikki nämä hajoamiset ovat (lähes) samanlaisia, vaikka ei osata laskea sitä, millaisia ne tarkalleen ovat.

Teorian mukaan B-mesoni siis hajoaa yhtä usein kaikiksi kolmeksi pariksi, ja parin jäsenet lentävät samoihin suuntiin, riippumatta siitä ovatko ne elektroneja, myoneja vai tauta antihiukkasineen.

Tänä keväänä julkistettujen CERNin LHC-kiihdyttimen LHCb-kokeen havaintojen mukaan kuitenkin B-mesoni hajoaa useammin elektroneiksi kuin myoneiksi. Jos Standardimalli pitää paikkansa, kerätyllä datamäärällä todennäköisyys tälle on 1:500. Kuten kokeellinen hiukkasfyysikko Tommaso Dorigo huomauttaa, tästä ei voi päätellä, että on tehty löytö 99.8% todennäköisyydellä. Koska kiihdytindatasta tarkistetaan satoja tai tuhansia erilaisia yhdistelmiä, sopii odottaakin, että vastaan tulee tällaisia tapauksia. Dorigo onkin valmis lyömään tuhat Yhdysvaltojen dollaria vetoa, että kyseessä ei ole uusi hiukkanen. Hiukkasteoreetikko Jester on sen sijaan varovaisen optimistinen.

Puheessaan Patel kuitenkin korosti sitä, että tämä ei ole ainoa B-mesonien hajoamiseen liittyvä poikkeama. B-mesoni hajoaa myoni-antimyoni-pariksi ennustettua harvemmin. Koska tässä hajoamistuotteissa ei ole mukana K-mesonia, tämän yleisyyden voi ennustaa tarkasti.

Kun laittaa poikkeamat yhteen, todennäköisyys sattumalle on naiivisti laskettuna 1:16 000, mikä alkaa olla vakuuttavaa. Lisäksi on havaittu, että syntyneiden hiukkasten suunnat poikkeavat Standardimallin ennusteista, mutta tämän kohdalla ennusteiden epävarmuus on toistaiseksi vielä iso.

Havainnoille olisi suhteellisen helppo selitys. Standardimallissa B-mesoni hajoaa samalla tavalla elektroneiksi, myoneiksi ja tauksi sen takia, että hajoamista välittävä Z-bosoni kytkeytyy samalla tavalla kaikkiin kolmeen hiukkaseen. Tämän voi muuttaa ottamalla mukaan uuden välittäjähiukkasen, joka vuorovaikuttaa eri tavalla näiden eri hiukkasten kanssa. Tällaiselle hiukkaselle on annettu mielikuvituksekas nimi Z’-bosoni. Toinen mahdollinen vaihtoehto on leptokvarkin nimellä tunnettu hiukkanen, joka muuttaa elektroneja, myoneja ja tauta kvarkeiksi ja toisinpäin (mikä on Standardimallissa mahdotonta).

Kuten tavallista, ongelmana on selittää, miksi muissa havainnoissa ei ole vielä näkynyt mitään. Jos kyseessä on uusi hiukkanen, sen massa on luultavasti hieman LHC:n nykyisten rajojen yläpuolella, eli se voi löytyä lähitulevaisuudessa. Mutta myös B-mesonien hajoamisesta saadaan lisää tietoa. Ehkä kiinnostavinta on se, että ensi maaliskuussa alkaa LHC:n kolmas havaintokausi, jonka aikana mitataan ensimmäistä kertaa B-mesonien hajoamista tau-antitau-pareiksi. Jos nyt esitetyt selitykset pitävät paikkansa, näiden hajoamisten pitäisi erota elektronihajoamisista vielä enemmän kuin myonihajoamisten.

Jotkut luonnehtivat viime kuussa julkistettuja tuloksia myonien magneettisesta momentista viimeiseksi toivoksi nähdä merkkejä Standardimallin tuonpuoleisesta hiukkasfysiikasta. Mesonien mittaukset osoittavat, että on olemassa toinenkin toivon reitti.

LHC:n käynnistyessä korostettiin sen törmäysten ennennäkemättömän korkeaa energiaa ja monet odottivat suuria löytöjä. Huomio kiinnittyi uusia hiukkasia etsiviin törmäyskokeisiin ATLAS ja CMS. Niiden dataa käytetään myös B-mesonien tutkimisessa, mutta nyt keskiöön on siirtynyt LHCb:n mittaamat pienet vaikutukset matalan energian tapahtumiin. B-mesonien hajoamisessa syntyneiden hiukkasen energiat ovat yli tuhat kertaa pienempiä kuin LHC:n protonitörmäyksissä. Aika näyttää, voittaako suti vasaran, vai jääkö Higgsin hiukkanen LHC:n ainoaksi löydöksi.

9 kommenttia “Suti ja vasara”

  1. Lentotaidoton sanoo:

    Niin tässä LHC:n tapauksessa hämminkiä ja sekaannusta yleisön silmissä on aiheuttanut sekä remontista että koronasta johtuneet viivytykset. Nythän LHC ei pyöri, eli nuo ilmoitetut tulokset perustuvat Run 2:n dataan. Tutkijoilla/teoreetikoilla on siis ollut hyvää aikaa syventyä ”vanhaan” dataan.

    Remontin jälkeinen aikataulu on venynyt eri syistä. Nyt Run3 alkaa maaliskuussa 2022 (kuten Räsänen sanoo), tosin koepyöritykset alkavat jo syyskuussa 2021. Tuolloin koko kone on aivan toista kuin Run kakkosen aikaan. Jos tämäkään ensi vuonna alkava kolmonen ei tuo selvyyttä mahdolliseen Standarditeorian ylityksiin niin saamme odottaa aina vuoteen 2027, jolloin totaalisti uudistettu High-Luminosity LHC (HL-LHC) alkaa operoimaan.

  2. Lentotaidoton sanoo:

    Sekä leptokvarkit että Z´bosonit näkyvät olevan oikea uusi ”tarha” mahdollisuuksia Standarditeorian ukjopuolelle. Ehkä tosiaan tulevat LHCn uudistukset antavat jotain suuntaa:

    Leptoquarks (LQs) are hypothetical particles that would interact with quarks and leptons. Leptoquarks are color-triplet bosons that carry both lepton and baryon numbers. Their other quantum numbers, like spin, (fractional) electric charge and weak isospin vary among theories. Leptoquarks are encountered in various extensions of the Standard Model, such as technicolor theories, theories of quark-lepton unification (e.g., Pati–Salam model), or GUTs based on SU(5), SO(10), E6, etc. Leptoquarks are currently searched for in experiments ATLAS and CMS at the Large Hadron Collider in CERN.

    In particle physics, W′ and Z′ bosons (or W-prime and Z-prime bosons) refer to hypothetical gauge bosons that arise from extensions of the electroweak symmetry of the Standard Model. They are named in analogy with the Standard Model W and Z bosons.
    Types of Z′ bosons
    Various models of physics beyond the Standard Model predict different kinds of Z′ bosons.
    Models with a new U(1) gauge symmetry
    The Z′ is the gauge boson of the (broken) U(1) symmetry.
    E6 models
    This type of model contains two Z′ bosons, which can mix in general.
    Topcolor and Top Seesaw Models of Dynamical Electroweak Symmetry Breaking
    Both these models have Z′ bosons that select the formation of particular condensates.
    Little Higgs models
    These models typically include an enlarged gauge sector, which is broken down to the Standard Model gauge symmetry around the TeV scale. In addition to one or more Z′ bosons, these models often contain W′ bosons.
    Kaluza–Klein models
    The Z′ boson are the excited modes of a neutral bulk gauge symmetry.
    Stueckelberg Extensions
    The Z′ boson is sourced from couplings found in string theories with intersecting D-branes .

    1. Martti V sanoo:

      Leptokvarkit esiintyvät tosiaan GUT teorioissa. Mielenkiintoista jos jotain sen ennustuksia saadaan LHC kokeilla esiin. Kokeet protonin hajoamisesta ei ole toistaiseksi tuottanut tuloksia.

  3. Martti V sanoo:

    Selkeä kirjoitus. Leptokvarkkien suora havainnointi taitaa olla ulottumattomissa. Nehän hajovat siis kvarkeiksi ja leptoneiksi ennen higgs kentän jäätymistä. Voitko avata hieman miten leptokvarkit selittäisivät merkinnässä esitettyjä tuloksia.?

    1. Syksy Räsänen sanoo:

      Standardimallissa sähkömagneettinen vuorovaikutus ja värivuorovaikutus eivät välitä reaktioita, missä tau, myoni tai elektroni muuttuisivat toisikseen. Heikon vuorovaikutuksen W- ja Z-bosonit tekevät niin. Ne kytkevät kaikkiin noihin kolmeen hiukkaseen samalla tavalla.

      Spekulatiivinen lepokvarkki tai X-bosoni välittää reaktioita, missä kvarkit ja leptonit muuttuvat toisikseen. Standardimallissa ei ole tällaisia reaktioita. Kuten Q- ja Z-bosonit, X-bosoni tyypillisesti kytkeytyy samalla tavalla kaikkiin kolmeen yllämainittuun hiukkaseen. Tällöin sen massan täytyy olla hyvin korkea, koska muuten se välittää reaktioita, missä protoni hajoaa koska yksi kvarkki muuttuu elektroniksi ja muiksi hiukkasiksi.

      Näitä havaintoja selittävissä malleissa on kolme perhettä leptokvarkkeja, yksi elektronille, yksi myonilla ja yksi taulle. Elektroneihin kytkeytyvän leptokvarkin massan pitää yhä olla iso, mutta myonin ja taun leptokvarkkien massat voivat olla pieniä. Havaintoihin sopii, että myoniin kytkeytyvän leptokvarkin massa olisi noin 40 TeV. Tällainen leptokvarkki muuttaisi b- ja s-kvarkkien ja myonien vuorovaikutusta sopivan määrän.

      1. Martti V sanoo:

        Kiitos hyvästä vastauksesta.

  4. Lentotaidoton sanoo:

    Eli tuo 40 TeV olisi pyöreesti 40 000 kertaa protonin massa! Aika jötikkä. Toisaalta tuo X-bosonin massa 10^15 GeV:kin on älytön eli 10^12 TeV. Taitaa insinööreillä olla monta mutteria kiristettävänä tulevaisuudessa.

  5. Antti sanoo:

    onko mitään uutta hiukkasta yleisessä etsinnässä samaan tapaan
    kuin oli Higgsin hiukkanen oli vuonna 2012?
    Eikö sillon ollut aika varma teoria että sellainen
    tulee vielä löytymään.
    Painovoima hiukkasen löytämiseen menee
    aikaa kymmeniä vuosia, veikkaisin, mutta
    onko mitään muita varteen otettavia hiukkasia
    tuloillaan?

    1. Syksy Räsänen sanoo:

      Useita uusia hiukkasia etsitään, mutta minkään olemassaolle ei ole samanlaista vankkaa pohjaa kuin mitä Higgsille oli. Lähimmäksi tulee pimeän aineen hiukkanen, mutta sen ominaisuuksista on toistaiseksi erittäin vähän tietoa havainnoista, joten ehdokkaat ovat hyvin erilaisia. Oma suosikkini on kevyet oikeakätiset neutriinot: https://www.ursa.fi/blogi/kosmokseen-kirjoitettua/kauneusvirheen-korjaaminen/

      Gravitoni on tapa kuvata yleisen suhteellisuusteorian tiettyjä piirteitä, ei hiukkanen samassa mielessä kuin hiukkasfysiikan hiukkaset. (Ja yksittäisen gravitonin havaitseminen niissä prosesseissa, joita niillä kuvataan, on tosiaan hankalaa.)

Vastaa

Sähköpostiosoitettasi ei julkaista. Pakolliset kentät on merkitty *

Ekologinen eläintarha

26.5.2021 klo 16.56, kirjoittaja
Kategoriat: Kosmokseen kirjoitettua

Kirjoitin kymmenen vuotta sitten mustista aukoista seuraavasti:

”Tällä hetkellä mustat aukot ovat tunnetun fysiikan rajalla: enemmän kuin pelkkiä arveluita, mutta vailla kiistatonta kokeellista varmennusta. Aihe yhdistää eri aloja: teoreettista ja kokeellista astrofysiikkaa, gravitaatioaaltojen fysiikkaa, hiukkasfysiikkaa ja suhteellisuusteoriaa. Tilanteessa on mielenkiintoisia teoreettisia mahdollisuuksia, ja kokeellinen tutkimus voi tarjota jotain sykähdyttävää.”

Neljä vuotta myöhemmin saatiin ensimmäinen suora havainto gravitaatioaalloista. Havaintoja on julkistettu nyt 50, ja suurin osa niistä vastaa hyvin yleisen suhteellisuusteorian ennusteita mustien aukkojen törmäyksistä. Jokusessa ainakin yksi parin jäsen on neutronitähti, ja muutamassa törmäilijöiden turhan pieni tai yllättävän korkea massa on herättänyt epäilyjä siitä, onko kyseessä sittenkään musta aukko tai neutronitähti, ainakaan tähden romahduksesta syntynyt.

Vuonna 2019 Event Horizon Telescope otti ensimmäisen valokuvan tapahtumahorisontin lähistöltä. Vuonna 2020 Nobelin palkinto meni (taas) mustille aukoille, teoreettisesta todistuksesta niiden muodostumisesta ja tarkoista havainnoista Linnunradan keskustan mustan aukon tienoilta.

Nyt on mustien aukkojen kulta-aika, ja niin tähtitieteilijät, kosmologit, kvanttigravitaatioteoreetikot kuin hiukkasfyysikotkin ovat kiireisiä aiheen parissa. Voisi siis luulla, että mustien aukkojen olemassaoloa ei enää juuri kyseenalaisteta. Asia on kuitenkin juuri päinvastoin. Koska on paljon uusia havaintoja, on mahdollista tarkemmin selvittää, onko kyse todella mustista aukoista.

Kappaleille, jotka näyttävät suunnilleen mustilta aukoilta on annettu nimi Eksoottinen Kompakti Objekti eli EKO (engl. Exotic Compact Object, ECO). Niitä on iso eläintarha, kuten fysiikassa vaihtoehtoisten kappaleiden tai hiukkasten kokoelmaa yleisesti kutsutaan. Eläintarha voidaan jakaa kahteen luokkaan.

Ensimmäisessä luokassa ovat kappaleet, jotka korvaavat mustat aukot. Yleinen suhteellisuusteoria ei ehkä kuvaa romahduksen lopputulosta täysin oikein, vaan se on yleisen suhteellisuusteorian pätevyysalueen ulkopuolella. Tällöin tarvitaan laajempi teoria gravitaatiosta –kuten säieteoria, silmukkakvanttigravitaatio tai jokin muu spekulatiivinen teoria– kertomaan mitä tapahtuu.

Yksi vaihtoehto on se, että kvanttifysiikka estää tapahtumahorisontin muodostumisen, ja tähti romahtaa loputtomasti, aina vain hitaammin. Toisaalta mustat aukot korvaavilla kappaleilla voi olla monimutkainen sisärakenne, joka vaikuttaa niiden käytökseen. Jacob Bekenstein oivalsi ja Stephen Hawking osoitti 1970-luvulla, että mustilla aukoilla on lämpötila ja entropia. Entropia on suure, joka mittaa sitä, montako toisistaan erotettavaa rakennuspalikkaa kappaleella on. Mustien aukkojen entropia herättää kysymyksen siitä, millainen niiden sisärakenne on.

Jotkut pitävät säieteorian suurena saavutuksena sitä, että siitä laskettiin mustilla aukoilla olevan juuri se entropia, minkä Bekenstein ja Hawking olivat päätelleet, ja selitettiin mitä niiden sisällä on. Lasku ei kuitenkaan kuvaa sellaisia mustia aukkoja, joita on oikeasti olemassa. Myös silmukkakvanttigravitaatiosta on laskettu sama entropia, mutta laskussa on tehty oletuksia, jotka eivät kaikkia vakuuta. On siis erilaisia ideoita siitä, mitä mustien aukkojen sisällä on, mutta asiasta ei ole varmuutta.

EKOjen toinen luokka ovat pienet ja tiheät kappaleet, jotka eivät korvaa mustia aukkoja vaan ovat olemassa niiden lisäksi. Ne eivät välttämättä synny tähtien romahduksessa, vaan aivan muulla tavoin. Tutkituin vaihtoehto on bosonitähdet.

Alkeishiukkasia on kahdenlaisia: fermioneita ja bosoneita. Tuntemamme aine koostuu fermioneista. Fermioneja ei voi olla kahta samassa tilassa. Tämän takia niistä voi pinota monimutkaisia rakenteita, ja atomit ja neutronitähdet pysyvät koossa.

Bosoneita voi sen sijaan olla samassa tilassa miten monta tahansa. Esimerkiksi laserissa on monta valohiukkasta samassa tilassa. Valo ei voi muodostaa rakenteita, koska se on massatonta ja liikkuu siksi valonnopeudella. Mutta jos on olemassa bosoni, jolla on massa ja joka ei hajoa nopeasti, niin sen hiukkaset voivat samassa tilassa yhtyä suureksi tähdeksi, joka on kuin jättimäinen alkeishiukkanen.

Bosetähdet voivat kenties räjähtää, ja tällaisen bosenovan voisi havaita. (Tässä näkyy fysiikan huumori: sana bosenova kuulostaa samalta kuin musiikkilaji bossa nova.) Toisaalta niiden törmäyksiä voisi havaita gravitaatioaaltojen kautta, mustien aukkojen tapaan. Bosonitähtien koko määräytyy hiukkasen massasta. Koska havaituissa gravitaatioaalloissa näkyy iso kirjo massoja, vain pieni osa niistä olisi mahdollista selittää bosonitähdillä.

Kummankin luokan EKOilla on neljä keskeistä ominaisuutta, jotka voivat olla erilaisia kuin mustilla aukoilla: koko suhteessa massaan, muoto, pinnan heijastavuus ja tapahtumahorisontti (tai sen puute).

On sikäli helppo etsiä merkkejä vaihtoehdoista mustille aukoille, että mustat aukot ovat hyvin yksinkertaisia kappaleita. Kaikki yksinäiset samanmassaiset ja yhtä nopeasti pyörivät mustat aukot ovat samanlaisia. Ne ovat symmetrisiä pyörimisakselin suhteen, ja samanlaisia kiertotaso ylä- ja alapuolella. Jos musta aukko ei pyöri, se on pallomainen. Lisäksi mustan aukon pinta ei heijasta lainkaan, vaan imee kaiken mikä siihen osuu, eikä sen sisältä tule mitään ulos, eli sillä on tapahtumahorisontti.

EKOt voivat pyöriessään ja toisiaan kiertäessään puristua eri tavoin, menettää energiaa sisäisen möyrinnän takia (aivan kuten tähdet) ja heijastaa osan gravitaatioaalloista.

Havaintojen tulkintaa vaikeuttaa se, että kahden mustan aukon järjestelmässä on paljon enemmän vaihtoehtoja kuin yhden. Esimerkiksi pyörimisnopeuden lisäksi pitää ottaa huomioon se, miten mustien aukkojen pyörimissuunnat suhtautuvat toisiinsa ja siihen tasoon, missä ne kiertävät toisiaan. Mitä useampi luku järjestelmää kuvaa on, sitä vaikeampi on selvittää sen yksityiskohtia gravitaatioaaltohavainnoista.

Nyt tehtyjen havaintojen perusteella voidaan kuitenkin jo sanoa, että jos EKOilla ei ole tapahtumahorisonttia, niiden säde on korkeintaan 10% mustaa aukkoa isompi. Myös EKOjen mahdollisesti mustista aukoista poikkeavia muotoja on mitattu, mutta niille ei ole saatu kovin tarkkoja rajoja.

Jos EKOn pinta heijastaa gravitaatioaaltoja, voi syntyä kaikuja. Jotkut ryhmät ovat väittäneet löytäneensä havaituista gravitaatioaalloista kaikuja, mutta tarkempien analyysien mukaan väitetty signaali ei erotu kohinasta.

Vuonna 2034 taivaalle nouseva gravitaatioaaltodetektori LISA pystyy selvittämään asiaa tarkemmin. Nykyiset gravitaatioaaltodetektorit LIGO ja Virgo näkevät vain noin sekunnin pätkän gravitaatioaalloista. LISA pystyisi seuraamaan samoja kohteita vuosien ajan ennen törmäystä, mikä lisäisi datan määrää valtavasti, vaikka signaali onkin hieman heikompi, koska aallot ovat voimakkaimpia kappaleiden kohdatessa.

Lisäksi LISAn odotetaan näkevän pieniä mustia aukkoja kiertämässä galaksien keskustojen jättiläismäisiä mustia aukkoja. Niissä on se vaikeus, että radat ovat erittäin monimutkaisia, eikä niitä ole vielä pystytty laskemaan edes supertietokoneilla. Ongelman kääntöpuoli on se, että koska radat ovat herkkiä mustien aukkojen ominaisuuksille, havainnot antavat tarkkoja rajoja ison mustan aukon ominaisuuksille kunhan ennusteet saadaan laskettua.

Toistaiseksi kaikki havainnot sopivat yhteen yleisen suhteellisuusteorian ennusteiden kanssa, eikä ole mitään luotettavaa ennustetta, jonka perusteella gravitaatioaaltokokeiden odottaisi näkevän merkkejä kvanttigravitaatiosta tai uudenlaisista tähdistä. Gravitaatioaallot ovat kuitenkin osoittaneet kykynsä luodata yksityiskohtaisesti niinkin äärimmäistä ilmiötä kuin tapahtumahorisonttia. Tulevat havainnot vähintäänkin lisäävät ymmärrystämme siitä, miten tarkasti yleinen suhteellisuusteoria kuvaa aika-avaruutta.

21 kommenttia “Ekologinen eläintarha”

  1. Cargo sanoo:

    ”Yleinen suhteellisuusteoria ei ehkä kuvaa romahduksen lopputulosta täysin oikein, vaan se on yleisen suhteellisuusteorian pätevyysalueen ulkopuolella. […] Yksi vaihtoehto on se, että kvanttifysiikka estää tapahtumahorisontin muodostumisen, ja tähti romahtaa loputtomasti, aina vain hitaammin.”

    Eikö asia hoituisi myös lisäämällä uusi valon nopeuden vakioisuuteen verrattavissa oleva klassinen periaate? Saataisiin siis analoginen tulos sille, että kappaleen liike-energia kasvaa rajatta, kun nopeus lähestyy valon nopeutta, eli tässä tapauksessa kappaleen energiatiheys kasvaa rajatta, kun avaruuden kaarevuus lähestyy jotakin kriittistä arvoa. Ja koska äärettömyydet ovat epäfysikaalisia, niin mitään todellista mustaa aukkoa ei voisi muodostua.

    1. Syksy Räsänen sanoo:

      Se, että valon nopeus on vakio ei ole fysiikan periaate. Se on tulos, joka seuraa sähkömagnetismin laeista.

      Se, että on olemassa suurin mahdollinen signaalinopeus on osa yleisen suhteellisuusteorian rakennetta, ei sekään erillinen periaate joka olisi lisätty erikseen.

      Tapahtumahorisonttiin ei liity ääretöntä kaarevuutta. Kaarevuus sen kohdalla riippuu mustan aukon koosta ja on sitä pienempi, mitä isompi musta aukko on. Mustan aukon keskustassa (yleisen suhteellisuusteorian mukaan) olevaan singulariteettiin liittyy ääretön kaarevuus, mutta sen muodostuminen on eri asia kuin tapahtumahorisontin muodostuminen.

  2. Lentotaidoton sanoo:

    Nuo bosonitähdet ovatkin sitten mielenkiintoinen lisä perinteiseen kosmologiaan/tähtitieteeseen – jos totta, niinkuin slogani kuuluu.

    Ilmeisesti mitkään nykyään tunnetut bosonit eivät käy. Monasti spekuloitu on esim axioni (jota myös ehdotetaan pimeän aineen hiukkaseksi). Mainitsit tuon Loop Quantum teorian: siinähän ehdotetaan Planckin tähteä eli Planckin energiatiheyden muodostamaa tähteä (jossa Heisenbergin epämääräisyysperiaatteen synnyttämä poisto”voima” pitäisi tähden kasassa).

    Mitä arvelet näistä?

    1. Syksy Räsänen sanoo:

      En ole Planck-tähdistä kuullutkaan.

      1. Lentotaidoton sanoo:

        In loop quantum gravity, a Planck star is a theoretically possible astronomical object that is created when the energy density of a collapsing star reaches the Planck energy density. Under these conditions, assuming gravity and spacetime are quantized, there arises a repulsive ”force” derived from Heisenberg’s uncertainty principle. In other words, if gravity and spacetime are quantized, the accumulation of mass-energy inside the Planck star cannot collapse beyond this limit to form a gravitational singularity because it would violate the uncertainty principle for spacetime itself

        https://en.wikipedia.org/wiki/Exotic_star

  3. Martti V sanoo:

    Kaikki tunnettu energia kvantisoituu riittävän pienessä mittakaavassa. Miksei myös aika-avaruus? Planckin tähti voi olla äärimmäinen tiheys. Singulariteetti oli jo Einsteinin mielestä ajatuksena vastenmielinen

    1. Syksy Räsänen sanoo:

      Tunnetun fysiikan mukaan kaikki energia ei ole kvantittunutta, ainoastaan sidottujen tilojen energia.

      1. Erkki Kolehmainen sanoo:

        Eikö kaikki energia muodostu kvanteista? Tämähän oli paradigman muuttanut Einsteinin idea, joka selitti myös valosähköisen iilmiön ja mullisti klassisen fysiikan perustan.

        1. Syksy Räsänen sanoo:

          Kuten yllä kirjoitin: ei. Einsteinin idea valon kvantittumisesta oli tärkeä askel kohti kvanttiteoriaa, ei osa kokonaista teoriaa. Kvanttimekaniikan koko teoria löydettiin 1920-luvun puolivälissä, ja se ennustaa täsmällisesti, milloin energia on kvantittunut ja milloin ei. (Ja tietysti myöhemmin löydetty kvanttikenttäteoria tekee niin myös.)

          Menee sen verta ohi merkinnän aiheesta, että ei tästä sen enempää.

  4. Martti V sanoo:

    Harmi ettei kvanttigravitaatiolla ole ennusteita. Gravitonin etsiminen lienee turhaa. Kvanntifysiikka varmasti esiintyy singulariteetissa, mutta siitä ei koskaan saada ennnustettavia havaintoja. Singulraariteetin romahdus kestäisi ikuisesti joten voidaan ajatella että niitä ei ole toistaiseksi syntyntyt.

    1. Syksy Räsänen sanoo:

      Ei singulariteetin syntyminen mustissa aukoissa yleisen suhteellisuusteorian mukaan kestä ikuisesti.

      1. Martti V sanoo:

        Ulkopuolisen havaitsijan näkökulmasta aikahan hidastuu loputtomasti

        1. Syksy Räsänen sanoo:

          Kun musta aukko on muodostunut, mustan aukon ulkopuolella sen suhteen paikallaan olevan ja sinne vapaasti putoavan kellojen käynnin ero on tosiaan ääretön putoajan ylittäessä tapahtumahorisontin. (Ulkopuolisen havaitsijan näkökulmasta putoaja ei siis koskaan saavuta tapahtumahorisonttia.)

          Tämä ei tarkoita sitä, että tapahtumahorisontin tai singulariteetin muodostuminen kestäisi äärettömän kauan.

  5. Seniorikosmologi sanoo:

    Mitä nämä blogit olisivatkaan ilman lukijoiden kommentteja ja nihin annettuja vastauksia! Tosin vastauksia en aina ymmärrä. Useankaan lukemisen jälkeen en oivalla, mitä tarkoittaa bloginpitäjän viimeisen vastaksen kohta: ”….mustan aukon ulkopuolella sen suhteen paikallaan olevan ja sinne vapaasti putoavan kellojen käynnin ero on tosiaan ääretön putoajan ylittäessä tapahtumahorisontin”? Voisiko tuon sanoa selkokielellä?

    1. Syksy Räsänen sanoo:

      Vastauksia ei ole aina kirjoitettu siten, että ne ovat ymmärrettäviä kaikille, ainoastaan kysyjälle, koska asian avaaminen voisi olla kokonaan oma merkintänsä.

      Mutta selitettäköön nyt. Gravitaatio vaikuttaa kellojen käyntiin. Mitä voimakkaampi gravitaatiokenttä, sitä hitaammin kellot käyvät. Tämän takia esim. GPS-satelliittien kellot käyvät nopeammin kuin kellot Maan pinnalla. Mustien aukkojen tapahtumahorisonttia lähestyttäessä tämä ero kasvaa rajatta. Kellon käynti riippuu myös havaitsijan liikkeestä. Vapaasti putoava havaitsija ohittaa tapahtumahorisontin äärellisessä ajassa, ulkopuolella paikallaan olevan havaitsijan kellon mukaan putoaja ei koskaan saavuta tapahtumahorisonttia, koska liikkuu aina vain hitaammin.

  6. miguel sanoo:

    Minulle sana ääretön herättää aina kysymysmerkin. Se on matemaattisesti yhtä fundamentaali kuin valonnopeus fysiikassa (mikä sekin riippuu väliaineesta). Mikä tahansa luku kerrottuna äärettömällä antaa vastaukseksi ääretön. Ei ole olemassa 0,1 x ääretön tai 2 x ääretön. Vastaus on aina ääretön, ei enempää eikä vähempää.

    Vaikka tämä ei liity blogin aiheeseen, niin esim. äärettömässä universumissa on jossain joku kolkka, jossa joku kirjoittaa samanlaisessa huoneessa samaa tekstiä kuin minä sama taulu seinällä. Jos se on mahdollista kerran, se on mahdollista toisenkin kerran, ja äärettömyydessä niin ei tapahdu kahdesti, vaan äärettömän monta kertaa. Ja kaikki muut vaihtoehdot toteutuvat myös äärettömän monta kertaa.

    Jos kellon käyntien ero on ääretön (Kun musta aukko on muodostunut, mustan aukon ulkopuolella sen suhteen paikallaan olevan ja sinne vapaasti putoavan kellojen käynnin ero on tosiaan ääretön putoajan ylittäessä tapahtumahorisontin), niin mihin se oikeastaan vastaa ja jos matematiikka antaa vastauksen ääretön, niin pitääkö fysiikan totella matematiikkaa?

    En pidä sanasta ääretön.

    1. Syksy Räsänen sanoo:

      Se, että jokin asia on ääretön ei tarkoita sitä, että se sisältäisi kaiken mahdollisen. Jos avaruus on ääretön (ei tiedetän onko näin), tämä ei tarkoita sitä, että kaikki mahdollinen tapahtuu jossain.

      Valonnopeus (eli suurin mahdollinen signaalinopeus) on eri asia kuin valon nopeus (nopeus jolla sähkömagneettiset aallot kulkevat) ks. https://www.tiede.fi/blogit/maailmankaikkeutta_etsim assa/luonnottomia_lokeroita

      Menee sen verta ohi merkinnän aiheesta, että ei tästä tämän enempää.

      1. MarttiV sanoo:

        Kiitos vastauksesta. Äärettömyys on toki hankala ymmärtää. Onko tapahtumahorisontin kohdalla aika-avaruudessa jonkinlainen epäjatkuvuuskohta, jos infidesimaalinen siirtymä aiheuttaa äärettömän hypyn kellon käymisessä? Sanoit, että ulkopuolisen havaitsijan näkökulmasta tapahtumahorisonttia ei koskaan saavuteta. Kun musta-aukkoon syöksyy materiaa, niin eikö havaintojen persuteella kuitenkin sitä päädy kuitenkin tapatumahorisontin sisälle?

        1. Syksy Räsänen sanoo:

          Ero kellojen käynnissä kasvaa jatkuvasti rajatta tapahtumahorisonttia lähestyttäessä.

          Ulkopuolisen havaitsijan näkökulmasta mikään kappale ei voi saavuttaa tapahtumahorisonttia äärellisessä ajassa. Mutta koska kappaleen lähettämä ja paikallaan olevan havaitsijan vastaanottama valo venyy rajatta kappaleen lähestyessä tapahtumahorisonttia, kappale häviäää näkyvistä äärellisen ajan kuluessa.

          Asiaa monimutkaistaa se, että kappaleet muuttavat mustan aukon tapahtumahorisonttia. Pienet kappaleet hyvin vähän, mutta kahden mustan aukon törmäyksessä niiden tapahtumahorisontit muuttuvat kokonaan ja sulautuvat yhteen.

  7. Joksa sanoo:

    Hawking itsekin päätyi lopulta epäilevälle kannalle mustien aukkojen suhteen. Tapahtumahorisonttikäsite tuntuisi ólevan perustavanlaatuisessa ristiriidassa myös gravitaatioaaltoilmiön kanssa. Nehän generoituvat ainakin osin käyttäen singulariteettien massaenergiaa, ja niiden välittämää informaatiota tarkkaillaan kiinnostuneina, myös mustien aukkojen sisäisen rakenteen arvioimiseksi. Vai olisiko niin että kosmologian informaatiokäsitteessäkin olisi selventämisen tarvetta?

    1. Syksy Räsänen sanoo:

      Singulariteeteilla ei ole mitään tekemistä mustien aukkojen törmäysten synnyttämien gravitaatioaaltojen kanssa. Singulariteetit ovat (oletettavasti) piilossa tapahtumahorisonttien takana, eivätkä vaikuta ulkopuolisen maailman tapahtumiin mitenkään.

Vastaa

Sähköpostiosoitettasi ei julkaista. Pakolliset kentät on merkitty *

Taide ja taivas

25.5.2021 klo 15.47, kirjoittaja
Kategoriat: Kosmokseen kirjoitettua , Kosmologia

Puhun tiistaina 8.6. joskus välillä kello 16-19 Taideyliopiston tilaisuudessa Taide ja tähtitaivas. Tilaisuus järjestetään Hietsun Paviljongissa Helsingissä. Sinne on ilmainen pääsy. Tapahtuman kuvaus on seuraava:

Taide ja tähtitaivas on taidetta, tähtitiedettä ja kulttuurihistoriaa yhdistävä keskustelutilaisuus, jossa puhutaan tähtikartoista, avaruuselokuvista ja maailmankaikkeuden epäinhimillisestä kauneudesta. Keskustelun alustajiin kuuluvat kulttuurihistorioitsija Maarit Leskelä-Kärki, kosmologi Syksy Räsänen, tähtitieteen dosentti Hannu Karttunen ja kuvataiteilija Elina Saloranta.

Puheeni otsikko on Kaikkeuden epäinhimillinen kauneus ja sen kuvaus on tämä:

Kun Aurinko vajaan kahdeksan miljardin vuoden kuluttua sammuu, ihmiskunta on kuollut sukupuuttoon kauan sitten. Silti Auringon loppu ja Maapallon tuho herättää surua. Ihmiset ovat tottuneet katsomaan maailmaa inhimillisen linssin kautta, joten epäinhimillisen maailmankaikkeuden tapahtumia verrataan ihmiselämään. Toisaalta ihmisille on kehittynyt tilan ja muotojen hahmottamiseen abstraktin ajattelun välineitä, joita on mahdollista soveltaa maailmankaikkeuden täsmälliseen ymmärtämiseen ja epäinhimillisen kauneuden kokemiseen.

Päivitys (28/05/21): Tilaisuus on koronarajoitusten takia siirretty tulevaisuuteen, keskiviikkoon 24.11. kello 16-19.

4 kommenttia “Taide ja taivas”

  1. Erkki Kolehmainen sanoo:

    Kaikkeuden epäinhimillinen kauneus tarkoittaa varmaan kaikkeuden jumalallista (tai saatanallista) kauneutta? Ihmettelen sitä, miten kauneus voi olla epäinhimillistä, koska kauneus on katsojan silmässä. Ettei vaan kaikki olisi kuitenkin suhteellsita. Surukin on sitä kuten nähtiin, kun jumalan valitseman kansan ilmavoimat moukaroivat surutta Gazan asuinkortteleiita ja Netanyahu jyrisi nyrkit pystyssä kuin suurikin profeetta!

    1. Syksy Räsänen sanoo:

      Gazaa pommitti Israelin ilmavoimat, ei ole juutalaisten ilmavoimia.

      Tästä osviittaa epäinhimilliseen kauneuteen: https://www.ursa.fi/blogi/kosmokseen-kirjoitettua/ihmisten-ja-enkelien-kielilla/

  2. ”Kun Aurinko vajaan kahdeksan miljardin vuoden kuluttua sammuu, ihmiskunta on kuollut sukupuuttoon kauan sitten. ” Tätä emme tiedä, koska se riippuu mm. siitä mitä ihmiset saavat päähänsä tehdä. Ihmislajin tulevaisuuden pituutta ei voi tietää ennalta, mm. koska ihminen pystyisi halutessaan muokkaamaan perimäänsä ja siten astumaan biologisen evoluution lakien ulkopuolelle.

    1. Syksy Räsänen sanoo:

      Vaikka näin kävisi, nuo olennot eivät olisi enää ihmisiä.

Vastaa

Sähköpostiosoitettasi ei julkaista. Pakolliset kentät on merkitty *


Kivikuvun alla

29.4.2021 klo 20.18, kirjoittaja
Kategoriat: Kosmokseen kirjoitettua , Kosmologia

Koeryhmän ANAIS edustaja Susana Cebrián Zarazogan yliopistosta Espanjasta puhui eilen Helsingin yliopiston kosmologiaseminaarien sarjassa viime kuussa julkistetuista tuloksista pimeän aineen jahdissa. Hän oli Helsingissä joulukuussa 2019 kertomassa ensimmäisistä mittauksista, ja antoi nyt etänä päivityksen, kun niiden tilastollinen merkitys alkaa olla huomattava.

Monet koeryhmät ovat etsineet pimeän aineen hiukkasta, mutta vain yksi väittää löytäneensä sen: DAMA/LIBRA. Kirjoitin yksityiskohdista täällä kolme vuotta sitten. Kokeen idea on yksinkertainen: eristetään koepala häiriöistä ja katsotaan, näkyykö siinä merkkejä törmäyksistä pimeän aineen kanssa.

Jos pimeää ainetta on olemassa, Maa kulkee pimeän aineen pilven läpi liikkuessaan Linnunradan keskustan ympäri. Tämän takia lävitsemme pyyhältää pimeän aineen hiukkasia noin 200 km/s nopeudella. Joskus pimeän aineen hiukkaset törmäävät tavallisen aineen hiukkasiin. Tämä on yksittäiselle hiukkaselle harvinaista, koska ne vuorovaikuttavat tavallisen aineen kanssa heikosti. Mutta jos katsoo tarpeeksi isoa kasaa hiukkasia tarpeeksi kauan eikä pimeän aineen vuorovaikutus ole liian heikko, niin näkee törmäyksiä.

Törmäys voi potkaista elektronin pois atomista, saada sen säihkymään valoa ja tärisyttää ainetta, jossa atomi on kiinni. On kuitenkin kaikenlaista kohinaa, joka saa aikaan tällaisia ilmiöitä muutenkin, kuten radioaktiivisten ydinten hajoaminen.

DAMA/LIBRA pyrkii erottelemaan signaalin kohinasta tarkastelemalla törmäysten vaihtelua vuoden aikana. Koska Maan nopeus Auringon ympäri on noin 30 km/s ja sen suunta muuttuu vuoden ympäri, pimeän aineen nopeus meidän suhteemme vaihtelee vuoden aikana noin 10%. Kun menemme pimeän aineen tuulta vastaan, pimeän aineen hiukkasia tulee vastaan tiuhemmin, ja myötätuuleen kulkiessa niitä tulee harvemmin.

DAMA/LIBRA näkee tällaisen vaihtelun vuodenaikojen myötä todennäköisyydellä, joka noin kaksikymmentä miljardia miljardia miljardia miljardia vastaan yksi. Maksimi on vieläpä juuri silloin kuin pimeältä aineelta odottaisi, 2. kesäkuuta. Tulos näyttää yksinään vakuuttavalta, mutta mikään muu koeryhmä ei ole onnistunut toistamaan sitä.

Kolme vuotta sitten totesin, että tämä ei ole ratkaisevaa, koska muut kokeet ovat käyttäneet erilaisia koemateriaaleja. On mahdollista, että pimeä aine vuorovaikuttaa eri tavalla eri alkuaineista koostuvan tavallisen aineen kanssa niin, että se näkyy DAMA/LIBRAn natriumjodidikristallissa, mutta ei esimerkiksi koeryhmän XENON1T nestemäisessä xenonissa.

Nyt tilanne on muuttunut. ANAIS käyttää samanlaista natriumjodidikristallia kuin DAMA/LIBRA, joten ei ole mitään syytä, miksei se näkisi samaa signaalia, jos kyseessä on pimeä aine. ANAIS-kokeessa on pyritty riippumattomasti muutenkin toistamaan DAMA/LIBRAn koejärjestely, ja huolella hallitsemaan virhelähteet.

Kokeessa on 112.5 kiloa natriumjodidikristallia suojakerrosten alla tarkkaan eristettynä Canfrancin laboratoriossa Pyreneiden vuoriston kivikuvun alla. On tärkeää sijoittaa koe kiven alle, koska taivaalta tuleva säteily on merkittävä kohinan lähde. Kun kosmiset säteet (eli nopeasti liikkuvat protonit ja muita kevyet ytimet) iskeytyvät ilmakehän atomeihin, syntyy hiukkassuihkuja, erityisesti myoneja, joita menee lävitsemme koko ajan. Myonien lukumäärä vaihtelee vuodenajan myötä: ilman tiheys riippuu lämpötilasta, ja mitä tiheämpää aine on, sitä enemmän kosmiset säteet siihen törmäävät.

Vaikka ANAIS on vuoren suojassa, koeryhmä erikseen mittaa myonien määrän. Samoin se seuraa kosteutta, painetta, lämpötilaa, ympäristön radioaktiivisuutta ja koepalan sisältä tulevaa radioaktiivisuutta ja muita ajan myötä muuttuvia tekijöitä.

Kokeen herkkyydestä kertoo jotain se, että yksi huomioitava virhelähde on se radioaktiivisuus, mikä syntyy kosmisten säteiden osuessa koepaloihin tai -laitteisiin matkalla niiden valmistuspaikasta laboratorioon. Tämä tosin vaimenee tilanteen rauhoituttua maan alla.

ANAIS aloitti datan keräämisen elokuussa 2017, ja koeryhmä on nyt käynyt ensimmäisen kolmen vuoden mittaukset. On helppo tiivistää tulos: ANAIS ei näe mitään signaalia. Todennäköisyys sille, että tämä on sattumaa ja DAMA/LIBRA on oikeassa, on noin yksi sadasta. Ryhmä kerää dataa vielä yhteensä kahden vuoden verran, jonka jälkeen todennäköisyyden odotetaan laskevan noin tekijään 1:400. Sen jälkeen ryhmä aikoo päivittää koelaitteistoa, jotta todennäköisyys saadaan niin alas, että DAMA/LIBRAn tulos on poissuljettu järkevän epäilyn tuolle puolen.

ANAISin lisäksi koe COSINE-100 on mitannut pimeän aineen mahdollisia törmäyksiä Etelä-Koreassa vuodesta 2016 alkaen. Se on julkistanut vasta ensimmäisen 1.7 vuoden mittausten tulokset, eivätkä ne riitä DAMA/LIBRAn näkemän vuodenaikojen vaihtelun varmistamiseen tai poissulkemiseen. Koe nimeltä SABRE pyrkii aloittamaan mittaukset samaan aikaan Australiassa ja Italiassa Gran Sasson kaivoksessa, missä DAMA/LIBRAkin on, käyttäen pallonpuoliskojen eri vuodenaikoja ympäristötekijöiden erottelemiseen. Gran Sassoon on menossa myös COSINUS, jossa ovat Helsingin yliopistolta mukana Matti Heikinheimom Alex Stendahl ja Kimmo Tuominen. Kaikki nämä kokeet käyttävät natriumjodidikristalleja, kuten DAMA/LIBRA.

Olen kosmologiaa luennoidessani käyttänyt DAMA/LIBRA-tuloksia esimerkkinä siitä, miten läpimurrot näyttävät usein selviltä vasta jälkikäteen. Tutkimuksen eturintamalla ei usein tiedetä, milloin yllättävät tulokset ovat merkkejä teoreettisen ymmärryksen puutteesta (tässä tapauksesta siitä, miten pimeän aine vuorovaikuttaa), ja milloin kyse on puutteista havainnoissa tai niiden tulkinnassa.  Yksi koe ei välttämättä riitä teorian varmistamiseen tai kumoamiseen, koska molempiin liittyy kaikenlaisia oletuksia, jotka otetaan tarkempaan syyniin vasta kun asiat eivät menekään odotetulla tavalla.

Vaikka DAMA/LIBRAn väitettyä pimeän aineen havaintoa ei ole vielä täysin tyrmätty, ANAIS on antanut sille aikamoisen iskun, ja harva enää odottaa sen nousevan kanveesista.

36 kommenttia “Kivikuvun alla”

  1. Mikko Valjakka sanoo:

    Nyt en ymmärrä jotain. Tunteehan pimeä aine painovoiman, eikö sekin siis ajaudu kiertämään Linnunradan keskustaa näkyvän aineen tavoin?

    1. Mikko Valjakka sanoo:

      ..vai onko kyse siitä, että se voi vuorovaikutuksen puutteen vuoksi kierrellä painovoimakeskusta täysin epäyhtenäisesti..?

      1. Syksy Räsänen sanoo:

        Aivan oikein. Näkyvä aine kiertää Linnunradan keskustaa yhtenäisesti, koska se vuorovaikuttaa itsensä kanssa (kaasupilvet työntävät toisiaan).

        Pimeän aineen vuorovaikutus itsensä kanssa on niin heikko, että se ei muodosta samanlaisia yhtenäisiä kokonaisuuksia, vaan hiukkaset kiertävät kukin minne sattuu. Niinpä hiukkaset ovat keskimäärin levossa Linnunradan kanssa, toisin kuin Aurinkokunta.

        Tämä ei ei täysin totta. Pimeä aine klimppiytyy kyllä gravitaation takia jonkin verran, ja siinä voi olla paikallisia virtauksia. Näiden vaikutusta pimeän aineen havaitsemiseen on tutkittu, ja yllä mainittu kuva on suurin piirtein totta.

  2. Cargo sanoo:

    Tuli mieleen, että jos pimeän aineen hiukkaset voivat muodostaa yhdisteitä, kuten näkyvä aine, niin silloinhan pimeiden hiukkasen havainnot/vuorovaikutukset voivat olla täysin mielivaltaisia. Onko siis mahdollista erottaa ”näkyvän ympäristön” satunnaishäiriöt ”pimeän ympäristön” häiriöistä, jotka voivat ilmeisesti olla myös satunnaisia?

    1. Syksy Räsänen sanoo:

      Pimeän aineen hiukkasten sidotut tilat on yksi mahdollisuus, jota on tutkittu DAMA/LIBRAn ja muiden kokeiden välisen ristiriidan selittämiseksi. Vaikka tämä onnistuisi, se ei selittäisi DAMA/LIBRAn ja ANAISin välistä ristiriitaa.

      Havainnot rajoittavat vahvasti sitä, miten voimakkaasti pimeä aine voi vuorovaikuttaa itsensä kanssa.

      Teorioista voi tehdä miten monimutkaisia tahansa, mutta jonkin pisteen jälkeen se ei ole enää hedelmällistä. Ks. https://www.ursa.fi/blogi/kosmokseen-kirjoitettua/edistys-ja-rappio/

  3. Eusa sanoo:

    Käsittääkseni mitään todentavaa tutkimusta ei ole siitä, että pimeän aineen halon liikemäärä olisi riippumaton tavallisesta aineesta – oikeastaan päinvastoin.

    https://arxiv.org/pdf/2104.10123

    Uutta tutkimusta siitä kuinka ”tidal field” on ehkä merkityksellistä kaikenkokoisissa haloissa.

    1. Syksy Räsänen sanoo:

      Tuo artikkeli käsittelee pimeän aineen simulaatioita. Ei siis pimeän aineen ja tavallisen aineen suhdetta, eikä havaintoja.

      Tämä riittäköön tästä.

  4. Eusa sanoo:

    Yleisen suhteellisuusteorian ratkaisuissa saattaa piillä vielä kovastikin vastauksia pimeän aineen luonteeseen.

    https://link.springer.com/content/pdf/10.1140/epjc/s10052-021-08967-3.pdf

    Tässä tuore laskelma frame-dragging-mekanismin mahdollisuuksista. Tulkinnanvaraisuuksia on suuntaan jos toiseenkin.

    1. Syksy Räsänen sanoo:

      Yleinen suhteellisuusteoria + tavallinen aine ei selitä havaintoja. Asia on järkevän epäilyn ulkopuolella.

      On mahdollista, että pimeää ainetta ei ole, vaan gravitaatiolaki on erilainen kuin yleisessä suhteellisuusteoriassa. (Kollegani Constantinos Skordis hiljattain juuri puhuikin aiheesta kosmologiaseminaarien sarjassamme.) Toistaiseksi ei kuitenkaan ole yhtäkään tällaista mallia, joka selittäisi kaikki havainnot, kun taas pimeä aine on selittänyt ja ennustanut havaintoja onnistuneesti.

  5. Kas sanoo:

    Onko olemassa näyttöä, että pimeän aineen pitää muodostua hiukkasista? Kysymyksellä tarkoitan sitä, että voiko pimeä aine olla itse avaruuden rakenteessa olevaa epätasaisuutta, ts avaruus olisi taipunut energiasta tai aineesta riippumatta.

    Älyllisesti ratkaisu olisi vähän laiska, koska kateissa olevan hiukkasen voisi aina selittää näin (muistuttaisi tietyllä tavalla multiuniversumi-teoriaa). Ja ovatko teoreettiset mallit (oletan että näillä olettamalla tehtyjä teorioita on rakennettu) merkittävästi nykyistä standardimallia/suhteellisuusteoriaa kompleksisempi?

    1. Syksy Räsänen sanoo:

      Kuten yllä kirjoitan:

      ”On mahdollista, että pimeää ainetta ei ole, vaan gravitaatiolaki on erilainen kuin yleisessä suhteellisuusteoriassa. (Kollegani Constantinos Skordis hiljattain juuri puhuikin aiheesta kosmologiaseminaarien sarjassamme.) Toistaiseksi ei kuitenkaan ole yhtäkään tällaista mallia, joka selittäisi kaikki havainnot, kun taas pimeä aine on selittänyt ja ennustanut havaintoja onnistuneesti.”

      Tarkemmin, ks. https://www.ursa.fi/blogi/kosmokseen-kirjoitettua/luodin-jaljet/

      1. Kas sanoo:

        Yllättävää, että ei ole löytynyt teoreettista mallia, joka periaatteellisella tasolla selittäisi poikkeamia, ei edes malleja, jotka ”väkisin runttaamalla” pystyisi nykyisiä havaintoja selittämään. Mutta tunnetusti mikään ei ole mahdotonta kun ei itse tarvitse tehdä.

        Joka tapauksessa näin maallikkona on erittäin mielenkiintoista seurata pimeän aineen tutkimusta. Kun ratkaiseva vinkki voi tulla niin monesta paikasta (hiukkaskiihdyttimet, gravitaatiomittaukset,, kaivoksiin rakennetut hiukkasten havaintomittarit yms) ja kun teoreettinen tutkimus ei ole löytänyt oikeita palasia (kenties matematiikka kehittynyt riittävälle tasolle). Toivottavasti ratkaisu kuitenkin löytyy seuraavan 50 vuoden aikana, olisi ikävä jos ei sitä pääsisi itse näkemään.

        1. Syksy Räsänen sanoo:

          On helppo ymmärtää, miksi on vaikeaa keksiä gravitaatiolakia, joka selittäisi kaikki pimeän aineen selittämät havainnot. Gravitaatio riippuisi tällöin vain tavallisen aineen jakaumasta. Pimeän aineen jakauma (joka selittää havainnot ja jonka mallit ennustavat) ei kuitenkaan seuraa näkyvän aineen jakaumaa. Pitäisi siis saada tavallisen aineen gravitaatiokenttä levittäytymään tavalla, joka ei noudata näkyvän aineen jakaumaa.

          1. Syksy Räsänen sanoo:

            Jos taas ajatellaan jonkinlaisia näkyvästä aineesta riippumattomia aika-avaruuden tihentymiä, niin ne ovat mahdollisia: toisin sanoen, pimeä aine voi koostua mustista aukoista.

  6. Jernau Gurgeh sanoo:

    Sabine Hossenfelder kirjoittaa uusimmassa postauksessaan, Backreaction -blogissaan, pimeästä aineesta ja modifioidusta gravitaatiosta.

    Hän luettelee nipun havaintoja, jotka puoltavat uutta gravitaatioteoriaa ja toisen nipun havaintoja, jotka puoltavat pimeää ainetta.

    Johtopäätöksenään hän esittää, että dikotomia näiden kahden välillä on virheellistä ajattelua ja yksi syy, miksi edistystä ei tapahdu.

    Tarvitaan sekä pimeää ainetta että muokattu gravitaatio, jotta kaikki havainnot voidaan selittää. Ongelma vain on, kuinka ne yhdistetään.

    Onko Syksyllä jotain näkökulmaa näihin puheisiin?

    1. Syksy Räsänen sanoo:

      Ei pidä paikkaansa, että havainnoista voisi päätellä, että molempia tarvitaan. Osa hänen mainitsemistaan pimeän aineen ongelmista eivät ole ongelmia, toisten kohdalla on epäselvää, miten pimeä aine ne selittää (vai selittääkö), koska galaksien dynamiikka on monimutkaista.

  7. Antti sanoo:

    voiko tietyt materiaalit emittoida pimeää ainetta ulos itsestään
    vähä niin kuin olisivat jollain tapaa epätasapainossa tai
    niihin vaikuttaisi jonkin ulkopuolinen energian lähde,
    vaikkapa kirjoituksessa mainittu painovoima.

    1. Syksy Räsänen sanoo:

      Tavallisen aineen sisällä ei ole pimeää ainetta. Jos pimeän aineen hiukkaset ovat hyvin kevyitä, niitä voisi syntyä ydinten radioaktiivisissa hajoamisissa. Tämän on kuitenkin pakko olla hyvin harvinaista, koska pimeää ainetta ei ole löytynyt. Lisäksi tällaisen kevyen hiukkasen vaikutus hajoamistuotteisiin olisi hyvin pieni ja vaikea havaita.

  8. Antti sanoo:

    ok. lukaisin tämän linkin ja siitä aikaisempi ajatus

    https://www.tekniikkatalous.fi/uutiset/pimean-aineen-teorioille-isku-auringon-ymparilta-puuttuu-se-mita-pitaisi-olla/94b2c66d-839c-301f-804d-5681fab6ef00
    ”Auringon ympäriltä puuttuu pimeää ainetta, vaikka sitä nykytietämyksen mukaan pitäisi olla. Teoriat galaksien liikkeistä voivat joutua romukoppaan, kertoo Euroopan eteläinen observatorio Eso.”

    entä pimeän aineen oskilloituminen? tieteisfiktiota?

    1. Syksy Räsänen sanoo:

      Juttu on maksumuurin takana, joten en kommentoi.

      Mitä tarkoitat termillä ”pimeän aineen oskilloituminen”?

  9. Antti sanoo:

    uutinen on 9 vuotta vanha ja koskee pimeän aineen puuttumista auringon lähellä,
    uudemmat teoriat on varmaankin paikanneet tämän puutteen tai asiaan on saatu selvyyttä
    muuta kautta.

    ”Mitä tarkoitat termillä ”pimeän aineen oskilloituminen”?”

    Onko ylipäätään mahdollista että pimeä aineen hiukkaset muuttuisivat
    muiksi hiukkasiksi tai toisinpäin?

    Onko sinulla osaa alla olevassa tutkimuksessa?
    https://www2.helsinki.fi/fi/uutiset/luonnontieteet/pimea-aine-pakenee-etsijoita-suomalaistutkijat-mukana-uudessa-jaljitysprojektissa

    1. Syksy Räsänen sanoo:

      Oskilloituminen on vain yksi erikoistapaus hiukkasten muuttumisesta toisikseen. Toki pimeän aineen hiukkaset voivat muuttua muiksi hiukkasiksi tai muut hiukkaset pimeän aineen hiukkasiksi. Esimerkiksi on mahdollista, että pimeä aine ei ole stabiili, jolloin se hajoaa joskus. Toisaalta voi olla, että maailmankaikkeudessa on yhtä paljon pimeää ainetta ja sen antiainetta, jolloin ne voivat annihiloitua muiksi hiukkasiksi.

      Ei. Mainitsen tuon projektin blogimerkinnässä yllä.

      Tämä riittäköön tästä.

  10. Martti V sanoo:

    Onko ollut ajatuksia että avaruus pyörisi galaksin mittakaavassa, jolloin tähtien kiertäminen keskustan ympäri ei olisi pelkästään gravitaation ehdoilla?

    1. Syksy Räsänen sanoo:

      Yleisen suhteellisuusteorian mukaan aine määrää aika-avaruuden ominaisuudet. Aine vaikuttaa avaruuden pyörimiseen, mutta vaikutus on hyvin heikko. Tämä on osa gravitaatiota. (Maapallon aikaansaama avaruuden pyöriminen saatiin mitattua vasta vuonna 2011.) Mikään näkyvän aineen gravitaatiovaikutus ei selitä näkyvän aineen liikkeitä.

      1. Cargo sanoo:

        Tuli mieleen sellainen crackpot idea, että voisiko pimeä aine olla jonkinlainen aika-avaruuden tiivistymä? Siis iso tilavuus avaruutta, jolla on jännitysenergiaa ilman mitään massaa tai säteilyä. Energia voisi siis lymyillä avaruuden rakenteessa ja jokin kosminen prosessi muodostaisi ko. tiivistymiä. Lisäksi voisi aprikoida, että tiivistymien jännitysenergia purkautuu ajan mittaan 🙂

        1. Syksy Räsänen sanoo:

          Kyllä, näitä kutsutaan mustiksi aukoiksi.

          1. Cargo sanoo:

            Mutta siis sellainen avaruudellinen alue, jolla on niin pieni energiatiheys, että fotonit sujahtavat läpi? Muutoin kyseinen alue voisi sitten toimia gravitaatiolinssin tavoin.

          2. Syksy Räsänen sanoo:

            Ei, fotonit eivät voi mennä läpi mustista aukoista.

          3. Cargo sanoo:

            Mutta voisiko avaruudella olla jonkinlainen hilarakenne, johon voi latautua jännitysenergiaa? Sellaiset tiivistymät olisivat kuin ohuita pilviä, joiden läpi tavallinen aine ja säteily singahtaa vailla sen suurempaa vuorovaikutusta.

          4. Syksy Räsänen sanoo:

            Yleisessä suhteellisuusteoriassa asia ei ole näin. Kaikenlaisia ideoita sen tuolta puolen toki on, myös avaruuden rakennetta on yritetty yhdistää pimeään aineeseen, mutta ei järin suurella menestyksellä. Tällaisia ideoita käytetään yleensä enemmän pimeän energian puolella (missä ne eivät myöskään ole olleet järin menestyneitä).

          5. Cargo sanoo:

            Entä voisiko osa alkuräjähdyksen energiasta, tai galaksien liike-energiasta, sitoutua gravitaatioaaltopaketeiksi, jotka sitten vaeltavat omaan tahtiinsa ja ilmenevät pimeänä massana? Salliiko yleinen suhteellisuusteoria hitaasti etenevien solitoneiden muodostumisen?

          6. Syksy Räsänen sanoo:

            Ei. Eiköhän tämä riitä tällaisista omista spekulaatioista.

      2. Martti V sanoo:

        Kiitos vastauksesta. Jos olisi riittävän suuri ja nopeasti pyörivä musta-aukko niin saako frame dragging pelkästään tähdet kiertämään sitä?

        1. Syksy Räsänen sanoo:

          Jos musta aukko pyörii tarpeeksi nopeasti, hyvin lähellä sitä on pakko kiertää mukana. Mutta kauempana massan vaikutus on merkittävämpi. Mutta menee sen verta kauas merkinnän aiheesta, että tämä riittäköön tästä.

  11. Antti sanoo:

    onko sinulla aihetta jossa käsitellään heikkoa vuorovaikutusta ja pimeää
    ainetta suhdetta? Kun sanotaan että että pimeä aine reagoi heikon vuorovaikutuksen
    kanssa niin koskeeko tämä reagointi sähköheikkoa teoriaa
    vai vaan tuota heikkoa vuorovaikutusta?

    1. Syksy Räsänen sanoo:

      Heikko vuorovaikutus on osa sähköheikkoa vuorovaikutusta.

      Pimeä aine voi vuorovaikuttaa heikon vuorovaikutuksen kautta, tai voi olla, että se ei tunne heikkoa vuorovaikutusta.

      Olen kirjoittanut heikon vuorovaikutuksen tuntevasta pimeästä aineesta täällä: https://www.ursa.fi/blogi/kosmokseen-kirjoitettua/nynnyjen-hautajaiset/

Vastaa

Sähköpostiosoitettasi ei julkaista. Pakolliset kentät on merkitty *


Mitä siis on aika?

13.4.2021 klo 17.27, kirjoittaja
Kategoriat: Kosmokseen kirjoitettua

Kirjoitin Helsingin opettajien ammattiyhdistyksen lehteen Rihveli ajan olemuksesta. Artikkeli alkaa näin:

Jotkut fyysikot tykkäävät siteerata Aurelius Augustinuksen (joka tuli tunnetuksi kirkkoisä Augustinuksena) teokseensa Tunnustukset 300-luvun lopulla kirjoittamia pohdintoja ajasta. Augustinus kommentoi osuvasti muun muassa sitä, miten vaikeaa ajan ajatteleminen on:

Mitä siis on aika? Jos kukaan ei kysy minulta, tiedän mitä se on. Jos haluan selittää sen sille, joka kysyy, en tiedä.

75 kommenttia “Mitä siis on aika?”

  1. MarttiV sanoo:

    Entropian etenemisen suunnalla ei taida olla tekemistä aika-avaruuden kaareutumisen kanssa

    1. Syksy Räsänen sanoo:

      Sitä ei tiedetä. Emme tiedä sitäkään, miksi signaaleja voi lähettää vain tulevaisuuteen, ei menneisyyteen.

      1. Cargo sanoo:

        Saako Nobelin palkinnon, jos ”kehittää” sellaisen painovoimateorian, jossa aika voi kulkea vain yhteen suuntaan?

        1. Syksy Räsänen sanoo:

          Jos se ei onnistuneesti ennusta mitään uusia havaintoja, niin ei. Nobelin palkinto myönnetään kokeellisesti varmennetuista läpimurroista.

  2. Fysiikan ajassa ei ole nykyisyyttä, menneisyyttä eikä tulevaisuutta. On vain kelloilla mitattavia tapahtumien kestoja joillakin jatkumoilla. Kuitenkin nykyisyyden muuttuminen menneisyydeksi ja uuden nykyisyyden aktualisoituminen tulevaisuudesta käsin on ajan fenomenologinen merkitys, se mitä Augustinuskin ehkä tarkoitti sanoessaan ymmärtävänsä mitä aika on vaikka ei osannut selittää sitä. Aika on olemassaolomme ydinrakenne ja siksi pakenee selityksiä.

    Fenomenologisen ajan ja fysiikan ajan välinen suhde on mielenkiintoinen filosofinen ongelma. On kuitenkin mielestäni selvää, että fenomenologista aikaa ei voi redusoida fysiikan aikaan eikä selittää sen avulla. Pikemminkin päinvastoin. Tämä kuvastaa ontologista näkemystäni yleisemminkin.

    1. Syksy Räsänen sanoo:

      Se mitä sanot fysiikasta ei ole totta, kuten artikkelissa kirjoitan.

      1. Satelliitti kiertää Maata. Aikaa kuluu, mutta missä on satelliitin havaitsijasta riippumaton nykyisyys? Kirjoitinko hieman epäselvästi? En näe ristiriitaa artikkelisi ja oman ajatukseni välillä. Täsmentäisitkö?

        1. Syksy Räsänen sanoo:

          Kuten artikkelissa kirjoitan, tämä ei pidä paikkaansa: ”Fysiikan ajassa ei ole nykyisyyttä, menneisyyttä eikä tulevaisuutta.”

          1. Erkki Kolehmainen sanoo:

            Kun näkee millaisessa umpikujassa hiukkasfysiikan standardimallin kannattajat ovat, niin ei heillä ole ainakaan tulevaisutta, vaikka vankka menneisyys ja hapertuva nykyisyys onkin.

          2. Syksy Räsänen sanoo:

            Asiahan on tismalleen päin vastoin: hiukkasfysiikan Standardimalli on ollut paljon odotettua menestyneempi, eikä hiukkaskiihdyttimissä ole 50 vuoden aikana löytynyt mitään varmistettua poikkeamaa siitä. Tämä riittäköön tästä, kun ei juuri liity aiheeseen.

  3. Jani sanoo:

    Onko aika havaitsijan havaitsemien havaintojen havaintojärjestys?

    1. Syksy Räsänen sanoo:

      Artikkeli toivon mukaan valaisee asiaa!

  4. Helena O sanoo:

    Minusta aika on kokemus, olemassaolon olemus. Jos aika on hypoteettinen tai teoreettinen (en löydä oikeaa sanaa) suure, se joka tapauksessa lakkaa olemasta, nimenomaan meille, kun meitä ei enää ole.
    Voi aika tietenkin olla muutakin.
    PS
    Löytyykö artikkelisi netistä?

    1. Syksy Räsänen sanoo:

      Artikkeli on linkattu blogimerkinnän sanassa ”Artikkeli”.

  5. Helena O: ”Minusta aika on kokemus, olemassaolon olemus.”

    Hyvin sanottu. Mielestäni ajan alkuperä on subjektiivisuudessa: nykyisyys, menneisyys ja tulevaisuus ovat mielekkäitä käsitteitä vain subjektiivisessa ajassa. Tosin myös objektien maailmassa voidaan puhua menneisyydestä ja tulevaisuudesta suhteesssa johonkin mielivaltaiseen aikakoordinaattiin, jota sitten kutsutaan nykyisyydeksi, mutta tämä on mielestäni käsitteiden väärinkäyttöä, tai ainakin pitäisi selventää mitä nykyisyydellä kulloinkin tarkoitetaan. On tietenkin mahdollista, että fysikaalisella nykyisyydellä on jokin sellainen merkitys, jota en ymmärrä, ja olisin utelias kuulemaan tai lukemaan siitä jonkin selityksen. Peräkkäisten tapahtumien välinen todennäköisyyssuhde ei mielestäni riitä luomaan nykyisyyttä maailmankaikkeuteen.

  6. Artikkelissa on lause ”Koska ei osata täysin sovittaa yhteen yleis-tä suhteellisuusteoriaa ja kvanttifysiikkaa, emme tiedä, onko tulevaisuus jo olemassa vaiko ei.”

    Mitä oikeastaan tarkoitetaan fysiikassa kun sanotaan jonkin olevan olemassa? MInusta selkeä määritelmä olisi että A on olemassa havaitsijan B suhteen jos B havaitsee A:n. Jos olemassaololla tarkoitetaan jotain laajempaa, esim. jos puhutaan tulevaisuuden tai horisontin takaisten asioiden olemassaolosta absoluuttisessa mielessä, joudutaan helposti siihen että olemassaolon käsitteestä tulee malliriippuva. Fysiikan käyttämissä matemaattisissa malleissa ei aina ole, eikä tarvitsekaan olla, vastinetta kaikille yleiskielessä käytetyille käsitteille.

    1. Syksy Räsänen sanoo:

      Tosiaan joskus hämärtyy, mitä olemassa oleminen oikeastaan tarkoittaa. Mutta tulevaisuuden olemassaolon ongelmaa voi havainnollistaa tuolla mnääritelmälläsi.

      Newtonilaisessa aika-avaruudessa voi ajatella, että joka hetkellä on olemassa avaruus, joka vain muuttuu ajassa.

      Suhteellisuusteoriassa kuitenkin havaitsija A voi olla saanut viestejä alueesta, mistä B:n ei ole ollut mahdollista ollut saada viestejä. Eli määritelmäsi mukaan toiselle osa aika-avaruutta olisi olemassa, toiselle ei. Ei ole yhteistä avaruutta, joten yleisen suhteellisuusteorian näkökulmasta on luonteva ajatella että koko aika-avaruus on olemassa.

      Kvanttifysiikan näkökulma on erilainen, kuten jutussa kirjoitan.

  7. Syksy Räsänen sanoo:

    Kuten artikkelissa kirjoitin, ajan luonnetta ei ole mahdollista saada selville tällaisilla arkiajatteluun pohjaavilla mietteillä, joten tämä riittäköön niistä.

  8. Erkki Kolehmainen sanoo:

    Voi herran pieksut ja rouvan lapikkaat, että asian voi tehdä vaikeaksi. Jo Nummisuutarin Eskolle (ja ehkä myös Valtaojalle) sanottiin, että maailma muuttuu Eskoseni. Ensin oli alkuräjähdys, sitten inflaatio ja pikku hiljaa alkoi muodostua hiukkasia. Aika on tätä muutosta ja sen vaiheita kuvaava termi. Sen yksiköksi kelpaa, mikä tahansa riittävän säännöllinen ja toistuva tapahtuma esim. atomiytimen värähtely tai kellon heilurin liike gravitaatiokentässä. Ja tähän muutokseen liittyy entropian kasvu eli muutos menee kohti tasapainoa ja on siksi koko maailmankaikkeuden mitassa peruuttamaton, vaikka paikallisia pienenmisiä voikin tapahtua.

    1. Syksy Räsänen sanoo:

      Tämä ei pidä paikkaansa: kuten artikkelissa kerrotaan, aika ei ole vain tapahtumien etenemisen yhtäläinen mitta.

      1. Erkki Kolehmainen sanoo:

        Vaikka aika ei ole yhtäläinen mitta, niin se itse kuvaa tapahtumien etenemistä eli aikayksiköiden vertaamisessa ei ole mitään järkeä.

        1. Syksy Räsänen sanoo:

          Aika ei ole vain tapahtumien etenemisen mitta, kuten artikkelissa selitetään. Tämä riittäköön tästä.

  9. MV sanoo:

    Voiko ajatella, että meillä on nopeus, jolla kuljemme eteenpäin ajassa, samoin kuin kuljemme kaiken aikaa tiettyyn suuntaan avaruudessa? Laajeneeko maailmankaikkeus muuten kiihtyvästi myös ajan suhteen, eli piteneekö aika, ja onko sitä mahdollista havaita?

    1. Syksy Räsänen sanoo:

      Kyllä, yleisessä suhteellisuusteoriassa havaitsijan nopeudella on neljä suuntaa: yksi aikasuunta ja kolme paikkasuuntaa.

      Niiden suuruus riippuu siitä, miten koordinaatisto valitaan (nopeus on suhteellista). Jos kiinnittää koordinaatit havaitsijaan, eli mittaa aikaa hänen kellonsa mukaan ja etäisyyksiä hänestä, niin havaitsija liikkuu ajassa eteenpäin valonnopeudella. (Kun aika-avaruuden kaarevuus jätetään huomiotta.)

      Ei, ajan kulku ei muutu avaruuden laajenemisen takia.

      1. MV sanoo:

        Kiitos! Onko syy siihen, että aika ei ”laajene”, jotenkin siis yhteydessä juuri valon nopeus -rajoitteeseen?

        1. Syksy Räsänen sanoo:

          Se, että informaatio ei voi kulkea valoa nopeammin ei liity tähän.

          Aika voi kulkea eri tahtia eri paikoissa (ja kulkeekin, vaikka ero pieni muualla kuin mustien aukkojen ja neutronitähtien läheisyydessä).

          Mutta aika ei voi tasaisesti hidastua tai nopeutua kaikkialla, koska merkityksellistä on vain ero kellojen käynnissä, Jos kaikki kellot käyvät yht’äkkiä eri tahtia, mikään ei muutu. (Noin karkeasti selitettynä.)

      2. Martti V sanoo:

        Matkustus menneisyyteen tekee maailmankaikkeuden epädeterministiseksi mutta myös epästabiiliksi ( esim isoisäparadoksi). Jos matkustus olisi mahdollista hiukkastasolla, voi sekin törmätessä johonkin toiseen hiukkaseen aiheuttaa ketjureaktion joka johtaa toisellaiseen tulevaisuuteen. Johtaako tämä siis multiversum ajatteluun? Jos ihminen pystyisi lähettämään signaalin ajassa taaksepäin se menisi rinnakkaiseen universuumiin

        1. Syksy Räsänen sanoo:

          Determinismi ei ole välttämättä ristiriidassa aikamatkailun kanssa.

          Determinismin kanssa voi kyllä tulla ongelmia, koska systeemin tila yhdellä hetkellä ei välttämättä määrää tulevaisuutta yksikäsitteisesti, jos on mahdollista vaikuttaa sekä menneisyyteen että tulevaisuuteen. Mutta voi myös olla, että määrää. On esitetty, että tämä olisi yksi ehto aikamatkailun mahdollisuudella, tai sen poissulkeva ehto. Ei tiedetä, miten asia on.

          Kvanttifysiikan kannalta tilanne on ongelmallinen, kuten yleisen suhteellisuusteorian aikakäsitys muutenkin, kuten artikkelissa kirjoitan.

  10. Jari Toivanen sanoo:

    Minä olen sitä mieltä, että aika on vain niin monimutkainen ja vaikea asia, että ihmislajin, edes älykkäimpien yksilöiden, järki ei riitä sen ymmärtämiseen, ei nyt eikä tulevaisuudessakaan. Olemme siis vain liian vajavaisia. En myöskään usko, että mikään tällä palstalla, tai missään muuallakaan, esitetty selitys ajan luonteesta on lähelläkään oikeata.

    1. Syksy Räsänen sanoo:

      Tämä ei ole mielipidekysymys sen enempää kuin aineen tai minkään muun fysikaalisen asian ymmärtäminen.

      Suhteellisuusteorian käsitys ajasta tekee ennusteita, jotka on onnistuneesti testattu suurella tarkkuudella. Esimerkiksi jos gravitaation vaikutusta ajan kulkuun ei ottaisi huomioon, GPS-paikannus ei toimisi.

  11. Jari Toivanen sanoo:

    En tarkoittanutkaan, että aika tai muutkaan fysikaaliset asiat an sich olisivat mielipidekysymyksia. Mielipiteeni koski ihmisen kykyä ymmärtää näitä asioita, tässä erityisesti ajan luonnetta. Voidaan mielestäni (taas mielipide) myös todeta, että koko ylläoleva keskustelu puoltaa mielipidettäni ajan luonteen ymmärtämisen haasteellisuudesta.

    1. Syksy Räsänen sanoo:

      Se, että jotkut eivät tunne jotain asiaa ei tarkoita sitä, etteikö kukaan tuntisi sitä – saati sitten, että sitä ei voisi tuntea.

  12. Jari Toivanen sanoo:

    Aivan, mutta siitä ei voi myöskään johtaa semmoista päätelmää, että välttämättä olisi joku, joka sen voisi tuntea.
    Jos tarkkoja ollaan, niin en väittänyt ”että sitä ei voisi tuntea”, vaan pelkästään sitä, että ihmisen kapasiteetti ei riitä sen tuntemiseen. En kiellä sitä, etteikö jossain voi olla älyllistä elämää, joka sen tuntee, tai että luonnonvalinta muokkaa ihmistä siihen suuntaan, että ajan ymmärtäminen olisi mahdollista.
    Siinä olet oikeassa, että mielipiteeni voi ajan kuluessa osoittautua vääräksi.

    1. Syksy Räsänen sanoo:

      Kuten yllä kirjoitin, koetulokset osoittavat, että olemme jo ymmärtäneet oikein paljon asioita ajan luonteesta, ja tällä ymmärryksellä on merkittävä teknologinen sovelluskin.

      Tämä riittäköön tästä.

  13. MarttiV sanoo:

    En tiedä miksi aiempaa kommenttia ei julkaistu. Voidaanko ajatella, että ajalla ei ole suuntaa ja se kuvaa etäisyyttä? Esim kahden hiukkasen välistä

    1. Syksy Räsänen sanoo:

      Yleisen suhteellisuusteorian mukaan aika on osa neliulotteista aika-avaruutta. Kahden hiukkasen välinen etäisyys voi olla ajankaltainen (eli niiden aikaetäisyys on pidempi kuin paikkaetäisyys), paikankaltainen (toisin päin) tai valonkaltainen (jolloin eikaetäisyys on yhtä iso kuin paikkaetäisyys).

      Aika-avaruuden etäisyydet määrittävät aika-avaruuden geometrian ja kaarevuuden, ja sitä kautta gravitaation.

      1. MarttiV sanoo:

        Onhan se luonnollista käyttää aikaa etäisyyksien mittaamisessa (esim. valovuosi). Energia kuten massa ”venyttää” aikadimensiota, mikä vaikuttaa valon kulkuun ja havaittuun etäisyyteen. Tämä venyminen aiheuttaa, että hiukkaset ovat aika dimensiossa kauempana toisistaan. Liittyykö tämä siihen, että aika kuluu suhteellisesti hitaammin esim. mustan aukon lähettyvillä?

        1. Syksy Räsänen sanoo:

          Yksiköiden valinta on sitten eri asia. Energia vaikuttaa aika-avaruuden kaarevuuteen. Tämän yksi ilmentymä on se, että aika kulkee massan lähellä hitaammin.

          1. Martti V sanoo:

            Paljonko on tutkittu gravitaation vaikutusta termodynamiikkaan tai kvanttimekaniikkaan? Esim koe avaruusessa voisi antaa eri tuloksia kuin maan päällä.

          2. Syksy Räsänen sanoo:

            Erittäin paljon. Kaarevan aika-avaruuden vaikutukset aineen kvanttifysiikkaan tunnetaan hyvin. Sen sijaan aika-avaruuden itsensä kvanttifysiikkasta ei tiedetä paljoa. Kosmisen inflaation parissa siitäkin on tehty ennustuksia joita on onnistuneesti testattu, mutta aika rajoitetussa määrin.

          3. Martti V sanoo:

            Sanoit että aika ”kulkee”. Jos aika on dimensio niin miten se kulkee tai sillä on suntaa? Termodynamiikalla on yksi suunta ja sen muutosnopeus johtuu gravitaatiosta

          4. Syksy Räsänen sanoo:

            Ilmaisu ”ajan kulku” viittaa siihen, miten avaruus ja siinä olevat kappaleet muuttuvat ajan edetessä.

            Ajalla on suunta siinä missä paikkasuunnillakin: eteen ja taakse. (Aika-avaruus on neliulotteinen kokonaisuus, jonka voi jakaa ajaksi ja paikaksi eri tavoin, joten tämä on vähän monimutkaisempaa, mutta en nyt selitä tarkemmin.)

            Termodynamiikan suunnan muutosnopeus (mitä se sitten tarkoittaakaan) ei liity gravitaatioon. Ei siitä sen enempää.

          5. MarttiV sanoo:

            Tarkoitin termodynamiikan muutosnopeutta, jolla järjestelmä menee kohti matalinta energiapotentiaalia ja epäjärjestystä, eikä sen suunta siis muutu. Esim kellosta energia siirtyy patterista kitkan kautta lämpöenergiaksi, mitä ei voi kääntää. Se miten nopeasti prosessi käy riippuu gravitaatiosta. Näin ollen kellolla mitattu sekuntti on johdannainen suure kyseisestä muutosnopeudesta ja luo arkikäsityksen ajasta. En näe siinä eriyistä mystiikkaa.

          6. Syksy Räsänen sanoo:

            Termodynaamisen kehityksen nopeus ei riipu gravitaatiosta, muuten kuin siten, että kaikkien mahdollisten tapahtumien tahti liittyy aika-avaruuden kaarevuuteen (koska ne ovat aika-avaruudessa).

            Tämä riittäköön tästä.

    2. Martti V sanoo:

      Lienee useimmille lukijoille selvä. Termodynamiikan kehitys vaikuuttaa käsitykseemme ajasta miksi tapahtumilla on tietty järjestys ja miksi on absurdia ”kulkea menneisyyteen”. Tämä sivuutettu

      1. Syksy Räsänen sanoo:

        Entropian kasvuun liittyvä ajan nuoli (jonka alkuperää ei tiedetä) ei ole ristiriidassa ajassa taaksepäin matkustamisen kanssa.

  14. Netissä paljon keskusteltu aikaan ja mustiin aukkoihin liittyvä paradoksi on tämä: Tapahtumahorisontissa aika pysähtyy meille ulkopuolisille tarkkailijoille. Kuinka siis mustaan aukkoon voi joutua ainetta, kun kaikki aine pysähtyy tapahtumahorisontin kohdalle? Eikö aine tarvitse aikaa siirtyäkseen jonnekin?

    Itse maallikkona mietin, että ratkaisu liittyy varmaan törmäävän kappaleen muodostaman aika-avaruuden ja mustan aukon muodostaman aika-avaruuden sulautumiseen toisiinsa ja tapahtumahorisontin siirtymiseen tämän seurauksena. Mikäköhän on oikea ratkaisu?

    1. Syksy Räsänen sanoo:

      Kysymys on sen verta kaukana merkinnän aiheesta, että ei siitä sen enempää. Yleisiä fysiikkaan liittyviä kysymyksiä voi lähettää esimerkiksi Tähdet ja avaruus -lehden kysymyspalstalle.

  15. Cargo sanoo:

    Jos ajan fysikaalinen olemus halutaan saada selville, niin millaisia peruskäsitteitä (epämääräisyys, entropia, energia, informaatio, valonnopeus jne.) tulee ottaa huomioon? Onko mahdollista että ajan ymmärtämiseksi tarvitaan jotakin vielä tuntematonta peruskäsitettä? Onko klassisella tai modernilla (fenomenologisella) filosofialla mitään roolia fysikaalisen ajan olemuksen selvittämisessä?

    Lisäksi tuli mieleen, että eikö myös suhteellisuusteoria ole kvanttimekaniikan ohella teoria informaatiosta, sillä ajan ja avaruuden lisäksi olennaista on äärellinen valonnopeus, joka taas kuvaa informaation ja vuorovaikutuksen välittymistä? Kuinka suuren osan fysiikasta voi selittää informaatio-käsitteen avulla?

    1. Syksy Räsänen sanoo:

      On harhaanjohtavaa puhua kuin aika joko ymmärrettäisiin tai sitten ei. Kuten fysiikassa yleensä, kysymys on ymmärryksen kasvusta.

      Suhteellisuusteoria ja kvanttifysiikka ovat molemmat tuoneet uudenlaisia käsitteitä. On luultavaa, että paremman ymmärryksen saavuttamiseen liittyy siihenkin uudenlaisia käsitteitä.

      Fenomenologisella filosofialla ei ole mitään annettavaa ajan fysikaaliseen ymmärtämiseen.

      Käsitys kvanttimekaniikasta teoriana informaatiosta on yksi idea, ei todeksi tiedetty asia. Fysiikkaa ei voi selittää informaation käsitteen avulla, vaan informaation käsite on yksi monista fysiikasssa.

  16. Tulee mieleen kaksi merkitystä menneisyyteen matkustamiselle:

    1. Lähdetään paikasta A vuonna 2021 ja palataan paikkaan A vuonna 2000.
    2. Lähdetään paikasta A vuonna 2021 ja saavutaan paikkaan B samanaikaisesti kun paikassa A eletään vuotta 2000.

    Jälkimmäinen tuntuisi loogisesti mahdolliselta, edellinen ei, mutta sitten tullaan samanaikaisuuden määritelmään ja sen ongelmiin.

    Kuten nähdään, olisi hyvä konkretisoida asioita ja väitteitä jos halutaan maallikoidenkin ymmärtävän fysiikkaa. Fyysikko ei ehkä tarvitse fenomenologiaa, mutta me muut tarvitsemme ainakin käsitteiden tuomista siihen mistä ne ovat lähtöisinkin: jokapäiväiseen elämään. En usko, että fysiikka on arkiajattelun tuolla puolen. Arkiajattelusta se on lähtenyt, enkä usko että yhteys siihen on voinut lopullisesti katketa. Oma työsi on siitä hyvä osoitus.

    1. Syksy Räsänen sanoo:

      Yleisen suhteellisuusteorian mukaan 1. on mahdollinen. (Ei tiedetä, onko se todellisuudessa mahdollinen.) Ks.

      https://www.tiede.fi/blogit/maailmankaikkeutta_etsimassa/ajan_kanssa

      https://www.tiede.fi/blogit/maailmankaikkeutta_etsimassa/ajankayton_hallinta

  17. Lentotaidoton sanoo:

    ”Entropian kasvuun liittyvä ajan nuoli (jonka alkuperää ei tiedetä)…”

    Käsittääkseni tässä se ”selitettävä” (toistaiseksi vain erilaisia spekulaatioita, monet liittyen esim sykliseen kosmokseen/kosmoksiin) asia on se, miksi entropian on kaikesta päätellen täytynyt olla ”alussa” (taas mitä se sitten oli) erittäin matala.

    Itse entropian kasvun ”ajan nuolihan” liittyy tietysti elämykselliseen kokemiseemme. Itse aikaahan se ei selitä.

    1. Syksy Räsänen sanoo:

      Ei sitäkään varsinaisesti tiedetä, miksi entropia aina kasvaa yhteen suuntaan (eli miksi ajassa liikutaan vain yhteen suuntaan). Koska tilastollisen fysiikan takana olevat klassisen fysiikan lait ovat symmetrisiä ajankäännössä, minkä tahansa todistuksen sille, että entropia kasvaa kun mennään ajassa eteenpäin voi muuttaa todistukseksi sille, että entropia kasvaa kun mennään ajassa taaksepäin vaihtamalla ajan etumerkin.

  18. Matias Slavov sanoo:

    Kiitos mielenkiintoisesta artikkelistasi. Minulla on yksi kysymys ja yksi kommentti.

    Jo suppean suhteellisuusteorian voidaan katsoa tukevan aikamuodotonta eternalismia (itse olen puolustanut tätä mm. täällä: https://link.springer.com/article/10.1007/s10701-020-00385-x). Jos QFT lukeutuu perustavimpien teorioiden joukkoon ja jos SR kuuluu siihen, niin eikö silloin SR:n implikoima käsitys ajasta (joka toki on kiistanalainen, mutta tyypillisesti sen katsotaan tukevan eternalismia) voi ajatella olevan hyvinkin perustavan tason kuvaus ajasta?

    Minusta vetoaminen kvanttifysiikan indeterminismiin ei osoita tulevaisuuden avoimuutta. Olkoon lait sitten deterministisiä tai probabilistisia, niin niistä ei ole löydettävissä mitään erottuvaa nykyhetkeä. Jos tiedämme hiukkasen tietyt muuttujat klo 12, Schrödingerin yhtälöllä voi laskea todennäköisyyksiä sen sijainnille klo 10 tai klo 14. Mikä on nykyisyyttä on meidän valitsema näkökulmamme, se ei ilmene itse laista. Laki ei kerro mitään siitä, onko ennen ajanhetkeä t jotain olemassa, tai onko ajanhetken t jälkeen jotain olemassa. Avoimen tulevaisuuden kannattajan on pidettävä nykyhetkeä (growing block teoreetikon mukaan myös menneisyyttä) jotenkin todempana kuin tulevaisuutta. Näin hänen on pidettävä samanaikaisuutta absoluuttisena. Kaiken avaruuden halkovaa ’nyt’-viipale on ristiriidassa samanaikaisuuden suhteellisuuden ja konventionaalisuuden kanssa.

    1. Syksy Räsänen sanoo:

      Suppea suhteellisuusteoria oli aikanaan perustavanlaatuisin teoria ajasta, sitten (kuten hyvin tiedät) tämän aseman sai yleinen suhteellisuusteoria, jonka aikakäsitys on hieman erilainen. Myös yleinen suhteellisuusteoria on vain approksimaatio jostain vielä kattavammasta teoriasta, joten sen käsitys ajasta tuskin pitää sellaisenaan paikkansa.

      Kvanttimekaniikkaan tuo erityisen tapahtumisen hetken aaltofunktion romahdus, jota Schrödingerin yhtälö ei kuvaa. On tietysti mahdollista, että aaltofunktio ei koskaan romahda, meistä vain näyttää siltä, mutta joka tapauksessa asiaa ei vielä ymmärretä.

      Kvanttimekaniikan samanaikaisuuden viipale on tosiaan ristiriidassa suppean suhteellisuusteorian kanssa, kvanttimekaniikassahan on samanlainen käsitys ajasta ja avaruudesta kuin klassisessa fysiikassa (romahdukseen liittyvää tapahtumista lukuun ottamatta). Kvanttikenttäteoriassa näin ei ole, mutta siinä tilan romahduksesta tulee suhteellinen. (Kuten EPR-paradoksin ratkaisussa.)

    2. Cargo sanoo:

      ”Jos QFT lukeutuu perustavimpien teorioiden joukkoon ja jos SR kuuluu siihen, niin eikö silloin SR:n implikoima käsitys ajasta (joka toki on kiistanalainen, mutta tyypillisesti sen katsotaan tukevan eternalismia) voi ajatella olevan hyvinkin perustavan tason kuvaus ajasta?”

      SR ei sisällä epätarkkuusperiaatetta, jonka mukaan menneillä ja tulevilla tapahtumilla on laadullinen ero. Eli jos hiukkasen paikka on joskus mitattu, niin se on realistinen osa todellisuutta, mutta mitään vastaavaa realismia ei ole mittauksen jälkeisessä tulevaisuudessa, ellei uutta mittausta suoriteta. QFT sisältää epätarkkuusperiaatteen eikä sen puitteissa mitään eternalismia esiinny.

      ”Jos tiedämme hiukkasen tietyt muuttujat klo 12, Schrödingerin yhtälöllä voi laskea todennäköisyyksiä sen sijainnille klo 10 tai klo 14.”

      Jos klo. 12 on olemassa tarkka informaatio esim. hiukkasen sijainnista, niin hiukkasen aaltofunktio tulee olla mitattu sijainnin suhteen klo.12. Kyseisessä mittauksessa aaltofunktio romahtaa ja aikakehitys alkaa ajanhetkestä nolla ja kulkee kohti tulevaisuutta. Siispä, jos hiukkanen sijaitsee (eli on objektiivinen osa todellisuutta) jossakin aika-avaruuden pisteessä klo.12, niin on mahdotonta arvioida sen sijaintia klo. ”miinus 2 tuntia”.

      ”Avoimen tulevaisuuden kannattajan on pidettävä nykyhetkeä (growing block teoreetikon mukaan myös menneisyyttä) jotenkin todempana kuin tulevaisuutta. Näin hänen on pidettävä samanaikaisuutta absoluuttisena. Kaiken avaruuden halkovaa ’nyt’-viipale on ristiriidassa samanaikaisuuden suhteellisuuden ja konventionaalisuuden kanssa.”

      Jos systeemiä kuvaava aaltofunktio on annettuna eikä systeemiä häiritä, niin tulevat ajanhetket kuvaavat jatkuvaa ja determinististä aikakehitystä. Mittaus on voimakas häiriö, joka antaa mitattavalle suureelle objektiivisen realismin; objektiivisuuden kannalta tuo mittaushetki on todellisempi kuin mittauksen jälkeiset ajanhetket. Mistään samanaikaisuuden universaalista absoluuttisuudesta ei voi tehdä päätelmiä, ja lisäksi sellaiset päätelmät vaatisivat ymmärrystä fysikaalisen ajan olemuksesta.

  19. Matias Slavov sanoo:

    Jos olen ymmärtänyt oikein, SR sopii hyvin kvanttifysiikkaan siinä missä GR ei. On siis hieman epäselvää, mikä teoria on perustavampi. Ehkä kysymykseni oli liian laaja tai epämääräinen tässä yhteydessä…

    Miten samanaikaisuusviipale oikeutetaan? Eihän meillä ole mitään keinoa varmentaa, että esim. nyt kirjoittaessani näitä lauseita Curiosity ottaa kuvan Marsissa. Tuo etäisyys on kosmisessa mittakaavassa mitättömän pieni. Kun edes noita kahta tapahtumaa ei saada yhdistettyä nykyisyyden hyperpinnalla, universaalin nykyhetken olemassaolo asettuu kyseenalaiseksi.

    En ymmärrä, miten probabilismin lisäksi esim. epätarkkuusperiaate tai mittausongelma auttavat presentistiä. Aikamuotoiset sijainnit ovat näkökulmia. Luonnonlaeissa tai maailmankaikkeudessa itsessään ei ole nykyisyyttä. Fysikaaliset tapahtumat ovat aikamuodottomasti aika-avaruudessa; ne itsessään eivät ole mennyttä, nykyistä tai tulevaa. ’Nyt’ on samanlainen indeksikaalinen ilmaus kuin vaikkapa ’tässä’. Nykyisyys ei ole sen todellisempaa kuin menneisyys tai tulevaisuus, kuten tässä oleminen ei ole sen todellisempaa kuin takana tai edessä oleminen. Yhden nykyisyys voi olla toisen menneisyyttä ja kolmannen tulevaisuutta. Tätä voi perustella melko yksinkertaisesti Lorentzin aikadilataatiolla SR:n kontekstissa ja gravitationaalisella aikadilataatiolla GR:n kontekstissa.

    1. Syksy Räsänen sanoo:

      Perustavanlaatuinen tarkoittaa sellaista, mitä ei (ainakaan toistaiseksi) osata johtaa mistääm muusta. Tässä mielessä suppea suhteellisuusteoria ei ole perustavanlaatuinen, koska sen voi johtaa yleisestä suhteellisuusteoriasta, joka on perustavanlaatuinen.

      Kvanttikenttäteoriaa ei osata johtaa mistään muusta, joten sekin on perustavanlaatuinen.

      Oletettavasti sekä yleinen suhteellisuusteoria että kvanttikenttäteoria ovat vain approksimaatioita teorista, jota ei ole vielä löydetty.

  20. leo sell sanoo:

    Eikö VOI ajatella niin että Makrotasolla on yhä se ajan nuoelen suunta pospäin alkuräjähdyksestä kute esim Sean Carroll ajan nuolen tutkijana tekee. Ja Mikrotasolla hiukkasilla on tuo aaltomaisuutensa kuten eletroniparikilla, jossa niiden aikakin voi hetken olla vastakkainen tai monipaikkainen.

    Siis että dekoherenssi tuottaa kaikkien kulkureittien summana yhä Feynmanin polkusummaintegraalin joka vastaa arkihavaintoamme siitä ettei särkynyt vaasi enää itsestään spontaanisti eheydy. Ja toisaalta muutamien fotonien tai elektronien tasolla on ajateltava kaikkien kulkureittien eri nopeus/aika eroja, jotka antaisivat otaksua sen pienen todennäköisyyden, jolla auto saadan autotalliin ”kaikkien hiukkasten” yhteisläpäisytodennäköisyyksiellä.

    Eli että makrotasolla yhä mittaamme aikaa jostain alkuhetkestä ja puhumme esim. maailmankaikkeuden iästä sen alusta lähtien kun taas äärimmäisellä mikrotasolla kaikki on vain aineaaltojen todennäköisyysamplitudeja olla samanaikaisesti monissa paikoissa, jolloin ajan suunnalla ei ole merkitystä ja aika voidaan jättää huomiotta. Olisiko kyse myös AIKA-käsitteen käytön mielekkyydestä näissä ääripäissä?

    1. Syksy Räsänen sanoo:

      Kvanttifysiikan aaltopaketteihin ei liity mitään vastakkaista tai monipaikkaista aikaa.

      Kaikenlaisia ideoita siitä, mitä on tuntemamme aikakäsityksen tuolla puolen on esitetty, emme tiedä mitkä (jos mitkään) niistä ovat oikeita.

  21. leo sell sanoo:

    Eli ilmeisesti oletat että edelleen on mielekästä käyttää tätä tuntemaamme aikakäsitystä. Mikä aikakäsitys kvanttifysiikassa vallitsee lomittumistapauksissa, joissa ennen parikkeina olleet spin-tilat ”kaukovaikutteisesti” samanikaisesti vaihtavat suuntiaan vaikkei valokaan ehtisi niille välittää tietoa tästä muutoksesta. Onko se sama kuin tämä valonnopeus-signaalin määrittämä aikakäsitys, tämä tuntemamme aikakäsitys? Ja mitä tarkoitat tuntemallamme aikakäsityksellä? Eikö niitä ole useampia?

    1. Syksy Räsänen sanoo:

      Kuten artikkelissa kirjoitan, tunnetun fysiikan puitteissa on kaksi (ristiriitaista) käsitystä ajasta ja tapahtumisesta.

      Kvanttimekaniikan käsitys ajasta on sama kuin klassisen fysiikan. Kvanttikentäteorian käsitys ajasta on sama kuin suppean suhteellisuusteorian. Lomittuminen ei ole asiassa oleellinen.

  22. leo sell sanoo:

    Hyvin vastasit, KIITOS siitä!

    Kuitenkin on olemassa ”matka” kuten Plancin pituus 10^-35/ Plancin ajalla 10^-44, joka on valon nopeus C. Eli matka ja aika tässä ovat molemmat mukana ja määrittävät toisiaan valon nopeuden kautta. Niillä ei kuitenkaan tämä tässä epäolennainen entanglementti – etäisyys eli matkan ylitys selity.

    1. Syksy Räsänen sanoo:

      Planckin pituudesta alla. Tämä riittäköön tästä.

      https://www.ursa.fi/blogi/kosmokseen-kirjoitettua/kaymattomista-korpimaista-vihoviimeinen/

  23. MarttiV sanoo:

    Ilmeisesti GR:n mukaan planckin mittakaava on pienin jakamaton aika-avaruuden rakenne ja olisi pistemmäinen. Jos sitä esittää jokin alkeishiukkanen, niin liittyykö siihen epämääräisyys myös aikadimensiossa?

    1. Syksy Räsänen sanoo:

      Yleinen suhteellisuusteoria ei ole kvanttifysikaalinen teoria. Siinä ei ole pienintä jakamatonta aikaa eikä paikkaa. (Kuten ei myöskään kvanttikenttäteoriassa.)

      1. Martti V sanoo:

        Aiemmasta merkinnästä käsitin että planckin mittakaava tulee gravitaatiosta. Plankin pituushan lasketaan gravitaatiovakiosta ja valonnopudesta

        1. Syksy Räsänen sanoo:

          Planckin mittakaava on yhdistelmä Newtonin (gravitaatio)vakiota, valonnopeutta ja Planckin vakiota, joista viimeksi mainittu liittyy kvanttifysiikkaan.

          1. MarttiV sanoo:

            Gravitaatiovakio on myös GR:ssä. Toki aika-avaruuden hiukkasluonnetta ei ole todistettu. Useimmat tutkijat taitaa kuitenkin pitää sitä todennäköisenä.

          2. Syksy Räsänen sanoo:

            Jos tarkoitat ”aika-avaruuden hiukkasluonteella” sitä, että olisi olemassa pienin pituus, en tiedä pitävätkö useimmat tutkijat sitä todennäköisenä. Se ei ole mikään lähellä tunnettuja asioita oleva idea, vaan kaukana tunnetusta oleva spekulaatio, joka voi pitää paikkansa tai olla pitämättä.

            Tämä riittäköön tästä.

  24. leo sell sanoo:

    Kohti ns. hierarkiaongngelman ratkaisua, kirjoittaa Masatoshi Yamada artikkelissaan jota luen. Olet blogissasi Käymätttömistä Korpimaista Viimeinen, kirjoittanut ASYMPTOOTTISESTI turvallisesta gravitaatiosta – tarkoittaako se eräiden Plancin vakioiden keskinäistä epäsuhtaa esim. että Plancin energiaa 1,22 x10^19GeV rajoittaa gravitaatiovakion koko, tällä rajaseudulla, jossa viimein päästäisiin käsiksi `lopullisiin luonnonlakeihin`. Sanot että ”gravitaation voima taittuu, jo ennen Plancin energiaa”. Eli onko tuossa HIERARKIAONGELMASSA kyse pikemminkin näiden ”ammoisten” vakioiden keskinäisestä epäsuhdasta, joka olisi renormalisoitava kohti esim. nykyisiä Higgsin kentästä saatuja arvoja kuten Higgsin massaan 125GeV liittyvä pituus 10^-18?

    1. Syksy Räsänen sanoo:

      Kysymys on sen verta kaukana merkinnän aiheesta, että ei siitä sen enempää.

  25. leo sell sanoo:

    Vastasit 29.4. ”kvanttifysiikan aaltopaketteihin ei liity mitään vastakkaista tai monipaikkaista aikaa”.

    Feynman kuvasi polku-integraaleissaan ja graafi-formalismissaan myös POSITRONEJA, siten että ajassa vastakkaiseen suuntaan liikkuvalla elektronilla voidaan kätevästi havainnollistaa positronien vastakkaista ”käyttäytymistä”. (signaalin voi myös nopeammasta eletronista lähettää fotonin vaihdossa hitaampaan matkalla A:sta B:hen ikäänkuin ajassa taaksepäin. Kaikkien mahdollisten reittien yhteenlaskussahan vasta saadaan lopullisen vaikutusamplitudin arvo. Tähän liittyy valonnopeuden ”keskiarvoisuus” . Tässä virtuaalisen fotonin vaihdossa kahden hiukkasen välillä se ei ole rajoitettu kulkemaan koko matkaa valon-norminopeudella – silloin se kulkiessaan hieman sitä nopeammin näyttäisi erityisen suhteellisuusteorian mukaan kulkevan myös ”ajassa taaksepäin”.

    Siten Feynmanin formalismissa vastakkaiseen suuntaan kulkeva elektroni voi kuvata positronia. Myöhemmin v.1949 Freeman Dyson osoitti, ettei tässä ”oikopolussa” antihiukkasiin ollut mitään mitä ei olisi voinut johtaa kvanttiteoriaan perustuvien yhtälöiden perussarjoista. Samalla kävi ilmi että hänen tuloksensa ja graafinsa olivat paitsi yhtäpitäviä Schwingerin ja Tomongan formalismien tulosten kanssa myös huomattavasti helpompia formuloida esim. mesonien tutkimuksiin soveltuviksi.

    1. Syksy Räsänen sanoo:

      Polkuintegraali on eri asia kuin aaltopaketti. Polkuintegraalit eivät myöskään liity informaation kulkuun liittyvään valonnopeuteen tuolla tavalla. Tämä riittäköön tästä.

Vastaa

Sähköpostiosoitettasi ei julkaista. Pakolliset kentät on merkitty *

Toivo elää

8.4.2021 klo 21.53, kirjoittaja
Kategoriat: Kosmokseen kirjoitettua

Eilen hiukkasfyysikot kohisivat kun tutkimuskeskus Fermilabin koeryhmä julkaisi uuden mittauksen myonin magneettisesta momentista. Lehdistötiedotteen mukaan tulos eroaa merkittävästi hiukkasfysiikan Standardimallin teoreettisesta ennustuksesta ja on vihje uudesta fysiikasta. Tässä on koeryhmän jäseneltä vähemmän hehkutusta ja enemmän tietoa sisältävä kirjoitus. Aiheesta on myös hyvä artikkeli Quantassa, ja teoreetikko Jester ja kokeilija Tommaso Dorigo valaisevat asiaa blogeissaan.

Myoni on alkeishiukkanen, eli sillä ei ole sisärakennetta. Alkeishiukkasten ominaisuudet tiivistyvät muutamaan numeroon: näitä ovat massa, sähkövaraus ja muihin vuorovaikutuksiin liittyvät varaukset, sekä jokunen muu luku. Yksi näistä on magneettinen momentti, joka kertoo, miten paljon hiukkanen kiertyy magneettikentässä. (Tämä kiertyminen liittyy hiukkasten ominaisuuteen nimeltä spin.)

Hiukkasen magneettisen momentin arvoon vaikuttavat kaikki hiukkaslajit, joiden kanssa se voi vuorovaikuttaa. Asiaa kuvaillaan usein niin, että hiukkasta ympäröi virtuaalisten hiukkasten pilvi, joka sekin reagoi magneettikenttään. Voi myös sanoa, että yksittäinen hiukkanen on hiukkaskentän tihentymä, ja liikkuu halki muiden kenttien, jotka vaikuttavat sen käytökseen.

Mittaamalla hiukkasen magneettisen momentin saa siis tietoa siitä, millaisia muita hiukkasia on olemassa. Elektronin magneettisen momentin laskeminen oli keskeinen osa kvanttielektrodynamiikan –ensimmäisen kvanttikenttäteorian– kehittämistä. Teoreettinen ja kokeellinen arvo täsmäävät miljoonasosan miljoonasosan tarkkuudella – tämä on yksi historian tarkimpia ennusteita ja mittauksia.

Myonien magneettinen momentti tunnetaan huonommin, ja Fermilabin koe on toistaiseksi tarkin. Koejärjestelyn idea on helppo selittää: myoneja kiihdytetään 99.9% valonnopeudesta ja pidetään voimakkailla magneeteilla kiertämässä kehää. Sekunnin sadastuhannesosassa myoni hajoaa elektroniksi ja neutriinoiksi. Kun mitataan, mihin suuntaan syntynyt elektroni osoittaa, saadaan selville mihin suuntaan myoni osoitti, eli miten paljon se oli kiertynyt magneettikentässä.

Fermilab on nyt määrittänyt myonin magneettisen momentin yli miljoonasosan miljardisosan tarkkuudella. (Lisätty tarkennus: koe mittaa myonin magneettisen momentin poikkeamaa siitä arvosta, mikä sillä olisi, jos myoni ei vuorovaikuttaisi muiden kenttien kanssa. Tämä poikkeama on noin tuhannesosan kokoinen, ja se on mitattu noin miljoonasosan tarkkuudella.)

Tämä ei ole iso edistysaskel: edellisen Brookhavenin laboratoriossa tehdyn mittauksen tarkkuus oli samaa luokkaa. Intoa herättää se, että nämä riippumattomat tulokset sopivat yhteen keskenään, ja eroavat teoreettisesta laskusta noin kolmen miljardisosan verran. Virherajaan verrattuna tämä on iso ero: todennäköisyys sille, että kyse olisi sattumasta on noin 1:40 000.

Hiukkasfysiikassa vaaditaan, että sattuman todennäköisyys olisi alle yksi noin miljoonasta, ennen kuin saa julistaa löytäneensä uutta fysiikkaa. Kuten olen usein maininnut, tilastollisen virheen korostaminen on kuitenkin harhaanjohtavaa, jos systemaattisia virheitä ja teoreettista ennustusta ei tunneta samalla tarkkuudella. Tässä tapauksessa ongelmia ei ole tiedossa niinkään kokeen kuin teorian puolella.

Fermilabin lehdistötiedote ei mainitse, että on olemassa toinenkin teoreettinen ennustus, ja se sopii hyvin yhteen mittaustuloksen kanssa. Ei tiedetä kumpi ennustus on oikein. On huvittava sattuma, että tämä yhteensopiva ennustus ilmestyi lehdessä samana päivänä, kun Fermilabin koetulos tuli julki. (Tulokset tosin luetaan nettiarkistosta arXiv, ei lehdistä, ja artikkeli on ollut siellä helmikuusta 2020 alkaen.)

Myonin magneettisen momentin laskeminen on erittäin vaikeaa. Siihen vaikuttavat monet hiukkaset, ja niiden vuorovaikutukset myonin kanssa pitää laskea tarkasti. Kaksi ristiriidassa olevaa teoreettista laskua lähestyvät asiaa eri tavalla.

Fermilabin koetuloksista poikkeava tarkastelu sivuuttaa osan laskujen ongelmista jättämällä niihin mustan laatikon, jonka arvo mitataan kokeista laskemisen sijaan. On mahdollista, että tämä laatikko ei ole asennettu oikein laskun muihin osiin.

Koetulosten kanssa yhteensopiva lasku sen sijaan laskee magneettisen momentin numeerisesti alusta alkaen. Numeerisissa laskuissa on omat sudenkuoppansa, ja laskun tehneen ryhmän mukaan tulos pitää vielä tarkistaa ja varmentaa.

Tilanne ei ole ainutlaatuinen. Muistan, kuinka 2000-luvun alussa oltiin kiinnostuneita myonin magneettisen momentin kokeellisen ja teoreettisen arvon isosta erosta, kunnes huomattiin, että laskussa oli yksi miinusmerkki väärin.

Fermilabin ryhmän julkaisema tulos perustuu vain 6% datasta, jonka se tulee kaikkiaan keräämään. Kaiken datan myötä tilastollisen virheen mahdollisuus laskee noin miljardisosan sadasosaan, kauas löydölle sovitun riman yläpuolelle, mutta tällä ei ole ratkaisevaa merkitystä, ellei teoreettista laskua saada vakaalle pohjalle.

Jos ristiriita koetuloksen ja teorian kanssa varmistuu, kyseessä on ensimmäinen kiihdytinhavainto hiukkasfysiikasta Standardimallin tuolla puolen. (Neutriinoiden massa ja pimeä aine ovat Standardimallin tuolla puolen, mutta ne on havaittu muilla keinoin.)

Kuten Jester huomauttaa, mitattu ero on isompi kuin Standardimallin W– ja Z-bosonien vaikutus myonin magneettiseen momenttiin. Tämä viittaa siihen, että poikkeaman aiheuttava hiukkanen ei ole kovin raskas. (Mitä raskaampi hiukkanen, sitä heikommin se vaikuttaa muiden hiukkasten ominaisuuksiin.) Tämä tarkoittaa, että sen voisi kenties havaita kiihdyttimissä – ja herättää kysymyksiä siitä, miksi sitä tai sen vaikutuksia muihin hiukkasiin ei ole vielä nähty.

Selityksiähän teoreetikoilla löytyy. Tänään tuli jo julki 32 artikkelia, joissa yhdistetään Fermilabin tulos uusiin hiukkasfysiikan teorioihin, ja huomenna saadaan lisää. Aika näyttää, onko joku niistä oikein, vai katoaako poikkeama tarkemmalla tarkastelulla, kuten hiukkasfysiikassa on usein käynyt. (Hiljattain julistettiin toinenkin –tilastollisesti vähemmän merkitsevä– ero Standardimallin ja havaintojen välillä; Tommaso Dorigo tarjoaa siihenkin hyvän katsauksen.)

Quantan artikkelissa Fermilabin koeryhmän jäsen Dominik Stöckinger sanoo liioitellen, että myonin magneettinen momentti on viimeinen toivo, ja hiukkasfysiikka kuolee, jos mitään ei löydykään. Tommaso Dorigo esittää asian vähemmän dramaattisesti: hänen mukaansa mittaus pitää elossa toivoa siitä, että on vielä jotain kiihdytinten löydettävissä.

Päivitys (10/04/21): Lisätty selvitys mittaustarkkuudesta.

20 kommenttia “Toivo elää”

  1. Martti V sanoo:

    Ilmiötä on spekuloitu viidentenä vuorovaikutuksena tai pimeänä energiana

    1. Syksy Räsänen sanoo:

      Tätä en ole huomannutkaan, mutta en ihmettele. Kaikenlaisia ideoita on!

  2. Ihan mahtava selkeytys! Kiitoksia. Nyt täytyy jännityksellä odottaa jatkokokeita ja teoreettisten kaavojen tarkistusta.

    1. Syksy Räsänen sanoo:

      Kiitos, mukava kuulla.

  3. Lasse Ilves sanoo:

    Momentti on määritetty yli miljoonasosan tarkkuudella. ja tulos poikkeaa kolmella miljardisosalla. Onkohan tuossa virhe kun miljardisosa on kai tuhannesosa miljoonasosasta ja tarkkuus oli siis tuota luokkaa eli havanittu ero paljon mittaus tarkkuuttaa pienempi.

    1. Syksy Räsänen sanoo:

      Kiitos, hyvä huomio. Ilmaisinkin tämän epäselvästi. Lisäsin tekstiin selvennyksen.

      Myonin magneettisen momentin mitattu arvo on g=2.00233184122. Jos myoni ei vuorovaikuttaisi muiden kenttien kanssa, g olisi tasan 2. Kokeissa mitataan myonin eroa tästä, eli arvoa (g-2)/2.

      Mittauksen tarkkuus suureelle (g-2)/2 on noin puoli miljoonasosaa, ja suureen (g-2)/2 koko on tuhannesosa. Näin magneettinen momentti siis tunnetaan miljardisosan tarkkuudella, vaikka mittaustarkkuus on vain miljoonasosa.

  4. Lentotaidoton sanoo:

    Vähän pelkään, että kysymyksessä voi olla BICEP2 tai OPERA – toisinnot. Mitä suuremmalla rytinällä tullaan julkisuuteen, sitä varmemmin tulisi varoituskellojen soida. Tosin ryhmä ilmoittaa jo aiemminkin saaneen tämänsuuntaista signaalia. Toisaalta taas todellisen datan laskennan suhde muuhun laskentaan arveluttaa.

    JOS tuo 4,2 sigma pitää (ja vielä ylittyy), niin toki seuraamme kiinnostuksella jatkoa.

    1. Syksy Räsänen sanoo:

      Fermilabin tulos on samanlainen kuin aiempi Brookhavenin mittaus, vähän tarkempi vain, eikä siinä ole isoja tiedossa olevia kysymysmerkkejä.

      BICEP2:n ja OPERA:n tulokset olivat uudenlaisia, eivät varmistuksia vanhalla. BICEP2:n kohdalla tuloksen tulkinnassa oli alusta alkaen kysymysmerkkejä, ja OPERA:n tuloksesta ei olisi koskaan pitänyt pitää lehdistötilaisuutta tai vihjata sen olevan löytö.

  5. Eusa sanoo:

    ”…on olemassa toinenkin teoreettinen ennustus, ja se sopii hyvin yhteen mittaustuloksen kanssa.”

    Mikä näiden ennustemallien olennaisin ero onkaan?

    1. Syksy Räsänen sanoo:

      Se on kerrottu tekstissä. Yksi korvaa osan teoreettisesta laskusta datalla, toinen laskee numeerisesti.

      1. Mikko sanoo:

        Eli voiko numeerisen mallin menetelmä sopia mittauksiin standardimallin mukaisesti ja selittää poikkeaman ilman uutta fysiikkaa?

        1. Syksy Räsänen sanoo:

          Kyseinen numeerinen lasku myonin magneettisesta momentista Standardimallissa on sopusoinnussa mittauksen kanssa. Niiden välillä ei ole mitään tilastollisesti merkittävää poikkeamaa selitettäväksi.

      2. Eusa sanoo:

        Onko tieteellisesti heikommalla pohjalla sellainen ennuste, joka nojautuu enemmän mittausdataan, vaikka teoreettinen malli olisi olemassa vai onko jopa löydettävissä teorian heikkouksia soveltamalla dataa sopivissa kohdin?

        1. Syksy Räsänen sanoo:

          Tässä ei ole kyse mistään periaatteesta. Kummallakin menetelmällä myonin magneettisen momentin laskemiseksi on vahvuutensa ja heikkoutensa.

  6. Erkki Kolehmainen sanoo:

    ”Myoni on alkeishiukkanen, eli sillä ei ole sisärakennetta.”

    Kuinka sitten on mahdollista, että myoni hajoaa elektroniksi ja neutriinoksi? Ja miksi neutriinoksi, eletronin vain myonin?

    1. Syksy Räsänen sanoo:

      Se, että alkeishiukkaset voivat muuttua hajota alkeishiukkasiksi ei ole ristiriidassa sen kanssa, että niillä ei ole sisärakennetta. Uudet hiukkaset eivät tule vanhan sisältä, vaan vanha hiukkanen muuttuu uusiksi.

      Myoni hajoaa elektroniksi, elektronin antineutriinoksi ja myonin neutriinoksi.

      1. Erkki Kolehmainen sanoo:

        Siis yhdestä leptonista syntyy kolme leptonia? Nyt tietysti herää kysymys. miten sisäinen rakenne määritellään ja mitä se ei voi olla?

        1. Syksy Räsänen sanoo:

          Alkeishiukkaset eivät koostu mistään pienemmistä osista. Jos hiukkanen koostuu joistakin pienemmistä osista, sillä on sisärakennetta.

          Enemmän täällä:

          https://www.tiede.fi/blogit/maailmankaikkeutta_etsimassa/sidottujen_kimppujen_vetovoima

  7. Jernau Gurgeh sanoo:

    Linkittyvätkö nämä mitenkään mahdolliseen myonien liian vähäiseen määrään B mesonien hajoamisessa, jota viimeisimmäksi on tutkittu LHCb:n toimesta?

    1. Syksy Räsänen sanoo:

      Riippuu täysin teoriasta. Jotkut teoriat varmaan pyrkivät selittämään molemmat poikkeamat osana samaa kokonaisuutta, mutta niillä ei välttämättä ole yhteyttä.

      B-mesonien mittauksissa havaitusta poikkeamasta, ks. merkinnän lopussa linkattu Tommaso Dorigon teksti:

      https://www.science20.com/tommaso_dorigo/another_3_sigma_fluke_from_lhcb-253707

Vastaa

Sähköpostiosoitettasi ei julkaista. Pakolliset kentät on merkitty *